Usmle part 1 part 15

DANIL HAMMOUDI.MD

SINOE MEDICAL ASSOCIATION

 


 

 

 

 

 

 

 

 

 

 

 

 

 


DIHYDROXYACETONE PHOSPHATE IS THE INTERMEDIATE PRODUCTS OF GLYCOLYSIS FOR FAT STORAGE.

 

Storage in the adipose tissue is catalysed by lipoprotein lipase, the activity of which is stimulated by insulin(the same hormone which stimulates storage of glucose as glycogen).

 

 ß-oxidation of fatty acids

 

This is a cyclic series of reactions (occurring within the mitochondria) with the end result of two carbon units being hydrolysed from the fatty acid chain with each cycle. These two carbon units are molecules of acetyl CoA.

With each oxidation cycle, a molecule of NAD is reduced to NADH and one FAD is reduced to FADH. These are re-oxidised by the electron transport chainwith the energy released coupled to ATP synthesis.

The acetyl CoA molecules formed in each cycle are oxidised to CO2in the citric acid cycle, with the oxidation/reduction reactions coupled to the electron transport chain and further ATP synthesis.

 

 

 

 

These events occur in liver and muscle. During sustained exercise the cells of slow twitch muscle fibres (which possess mitochondria) utilise ß-oxidation as the major source of ATP.

 

 

 

 

Ketone bodies

 

An alternative method of utilising the acetyl CoA formed by ß-oxidation is via the synthesis and subsequent oxidation of four-carbon units known collectively as ketone bodies.

Acetyl CoA is converted in the liver into acetoacetate (essentially two acetyl groups covalently linked). Acetoacetate can be further reduced to form ß-hydroxybutyrate. These two compounds are referred to as ketone bodies. Their synthesis occurs in the liver.

They diffuse from the liver into the circulation and are used as fuels by several tissues. Heart muscle and renal cortex, in particular, use acetoacetate in preference to glucose. In contrast, glucose is the major fuel for the brain and erythrocytes in a human on a balanced diet. The brain has the capacity to adapt to the use of acetoacetate during starvation (and in the metabolic disease diabetes mellitus). In starvation of long standing, acetoacetate meets more than 70% of the energy needs of the brain.

 

This ability of the brain to adapt to the use of acetoacetate is important because fatty acids cannot enter neural tissue. Acetoacetate is regarded as a water soluble and readily transported form of acetyl CoA.

The efficiency (amount of ATP produced) of oxidation of fatty acids directly or via formation of ketone bodies is approximately the same - there is no penalty to the body in converting acetyl CoA to this water soluble form.

It is important to be aware thatthere is no mechanism in animals for the conversion of fatty acids to glucose.

 

 

 

 

Fatty acid synthesis

 

This occurs in the cytoplasm of cells (compared to ß-oxidation which occurs inside the mitochondria).

The process begins with acetyl CoA and cyclic reactions join two-carbon units to the growing fatty acid chain. The completion of the synthesis and the formation of unsaturated fatty acids is complex.

The role of fatty acid synthesis is to :

supply the body's needs for particular fatty acids not supplied in the diet AND

 

 

to convert excess dietary glucose to fatty acids for storage

 

 

Glucose is converted to pyruvate (glycolysis), then to acetyl CoA which, when ATP is required, is oxidised by the citric acid cycle. If the glucose intake exceeds the body's energy needs (and after saturation of glycogen stores) the acetyl CoA can be used for fatty acid synthesis (in the liver) and storage as triglyceride in adipose tissue.


A newborn presents with severe acidosis, vomiting, hypotonia, and neurologic deficits. Serum analysis reveals elevated levels of lactate and alanine. These observations suggest a deficiency in which of the following enzymes?

A. Alanine aminotransferase
B. Glutamate dehydrogenase
C. Lactate dehydrogenase
D. Pyruvate carboxylase
E. Pyruvate dehydrogenase

 

A group of patients with lung cancer is matched to a group of patients without lung cancer. Their smoking habits over the course of their lives is compared. Based on this information, researchers compute the rate of lung cancer in patients who smoke versus those who never smoked. This is an example of

A. case-control study
B. cohort study
C. cross-sectional study
D. longitudinal study
E. randomized control study

A pharmacologist is examining a new drug with potential sedative properties. He begins by analyzing the pharmacokinetic properties of the drug. Studies of the drug's rate of elimination yield the above data. Which of the following drugs has similar kinetics to the drug being studied?

A. Carbamazepine
B. Cimetidine
C. Ethanol
D. Ketoconazole
E. Phenobarbital

 

 

A 46-year-old woman presents to her doctor complaining of weakness and fatigue. On physical examination, her physician notices a 10-pound weight gain since her last visit 6 months ago. Her blood pressure is 160/100 mmHg. Blood tests reveal serum Na+ 155 mEq/L, K+ 2.8 mEq/L, and a decreased serum renin. Which of the following is the most likely diagnosis?

A. Cushing's syndrome
B. Diabetes mellitus
C. Pheochromocytoma
D. Primary aldosteronism
E. Secondary aldosteronism

 

A 38-year-old woman complains of cold, painful fingertips, as well as difficulty swallowing and indigestion. Physical examination is remarkable for a thickened, shiny epidermis over the entire body, with restriction of movement of the extremities, particularly the fingers, which appear claw-like. Which of the following autoantibodies is likely to be found in this patient's serum?

A. Anti-DNA topoisomerase I (anti-Scl-70)
B. Anti-double-stranded DNA (ds DNA)
C. Anti-IgG
D. Anti-Sm
E. Anti-SS-A

 

A 26-year-old woman and her 29-year-old husband have been trying to have a child for the last 3 years. During this time the woman has had 5 spontaneous abortions. The karyotypes of the mother, father, and the most recently aborted fetus all contained 46 chromosomes, and all pairs were normal except for the pairs shown below.



The event that led to the child's abnormal karyotype was:

A. adjacent I segregation
B. adjacent II segregation
C. alternate segregation
D. a recombination event within a paracentric inversion
E. a recombination event within a pericentric inversion

 

A 70-year-old woman undergoes a gastrectomy for Zollinger-Ellison syndrome. Her doctor informs her that she will need to take intramuscular vitamin B12 shots for the rest of her life. Absence of which of the following cell types is responsible for this vitamin replacement requirement?

A. Chief cells
B. G cells
C. Goblet cells
D. Mucous neck cells
E. Parietal cells

 

A patient presents with right lower quadrant pain, fever, and diarrhea. Physical examination reveals diffuse abdominal tenderness; laboratory examination reveals a moderate leukocytosis, leading to a presumptive diagnosis of acute appendicitis. Surgical exploration of the abdomen reveals mesenteric adenitis, but the appendix is normal. Which of the following organisms is most likely responsible for these signs and symptoms?

A. Clostridium difficile
B. Enterohemorrhagic E. coli
C. Enteroinvasive E. coli
D. Enteropathogenic E. coli
E. Yersinia enterocolitica

 


Mean arterial blood pressure (MAP) and heart rate (HR) measurements were recorded during the intravenous administration of two different drugs. Select the most likely drugs given at the indicated points.

A. X, Acetylcholine Y,phentolamine
B. X, epinephrine Y,hexamethonium
C. X, isoproterenol Y, propranolol
D. X, metaproterenol Y, propranolol
E. X, norepinephrine Y, hexamethonium

 

A 21-year-old man is competing in a weight-lifting competition. He lifts 325 lbs over his head and holds it there for 5 seconds. Suddenly, his arms give way and he drops the weights to the floor. Which of the following receptors is responsible for this sudden muscle relaxation?

A. Free nerve ending
B. Golgi tendon organ
C. Merkel's disk
D. Muscle spindle
E. Pacinian corpuscle

 

A normal volunteer consents to an intravenous infusion of p-aminohippuric acid (PAH). After a short time, the plasma PAH is 0.02 mg/ml, the concentration of PAH in urine is 13 mg/ml and the urine flow is 1.0 ml/min. What is the effective renal plasma flow?

A. 0.26 ml/min
B. 26 ml/min
C. 65 ml/min
D. 260 ml/min
E. 650 ml/min

 

A 4-day-old male is brought to the pediatric clinic because of breathing difficulties and poor feeding. He coughs, chokes, and spits up milk very soon after beginning to suckle. Physical exam and radiographs reveal the presence of the most common type of tracheoesophageal fistula. The baby's defect likely resulted from:

A. failure of the buccopharyngeal membrane to rupture
B. failure of the tracheoesophageal ridges to fuse
C. incomplete formation of the septum secundum
D. incomplete recanalization of the larynx
E. patent thyroglossal duct

 

A 4-month-old blue-eyed, fair-skinned child who appeared normal at birth is brought to the pediatrician because of the development of scaly skin lesions and seizures. On physical examination, the infant appears to be mentally retarded, and a musty odor is noted. Which of the following defects is responsible for this child's illness?

A. Deficiency of alpha-ketoacid decarboxylase
B. Deficiency of hexosaminidase A
C. Deficiency of homogentisate oxidase
D. Deficiency of hypoxanthine-guanine phosphoribosyltransferase
E. Deficiency of phenylalanine hydroxylase

 

A 31-year-old stockbroker drives to a high-altitude mountain resort to do some rock-climbing. Later that day, he experiences headache, fatigue, dizziness, and nausea. Which point on the graph above best corresponds to the relationship between plasma bicarbonate, pH, and PCO2 in this patient?

A. Point A
B. Point B
C. Point C
D. Point D
E. Point E

 

A 13-year-old male presents to the emergency room with a deep skin abrasion on his knee. He states that it has not stopped bleeding since it happened during recess approximately 20-30 minutes ago. Physical examination reveals a well-developed, well-nourished adolescent. There are multiple purpura over his legs and arms, and a few scattered petechiae on his chest and gums. His bleeding time is 22 minutes, platelets = 300,000/mm3, hemoglobin = 11g/dL. A trial of cryoprecipitate transfusion does not improve his bleeding time. A normal platelet transfusion does improve bleeding time. Which of the following is the correct diagnosis?

A. Bernard-Soulier syndrome
B. Henoch-Schönlein purpura
C. Idiopathic thrombocytopenic purpura
D. Thrombotic thrombocytopenic purpura
E. Von Willebrand's disease

 

An 88-year-old male complaining of abdominal pain enters the emergency room with his wife. A mini-mental status exam reveals pronounced forgetfulness and confusion. The patient is discovered to have acute appendicitis requiring immediate surgery. He is unable to understand the situation and cannot provide informed consent. Which of the following further actions must the physician take?

A. Do not perform surgery
B. Have another doctor confirm the necessity of surgery
C. Obtain a court order to perform surgery
D. Obtain consent from his wife to perform surgery
E. Try to persuade the patient to consent to surgery

 

A 42-year-old obese woman experiences episodic abdominal pain. She notes that the pain increases after the ingestion of a fatty meal. The action of which of the following hormones is responsible for the postprandial intensification of her symptoms?

A. Cholecystokinin
B. Gastrin
C. Pepsin
D. Secretin
E. Somatostatin

 

Which of the pharyngeal pouches develops into the palatine tonsil?

A. First
B. Second
C. Third
D. Fourth
E. Fifth

A worried mother complains to her pediatrician that both she and her 6-year-old son's teacher have noticed that the child has become inattentive. She states that her son frequently stops what he is doing and "stares blankly into space" before resuming his activities. Electroencephalography reveals a 3/second spike and slow wave pattern of discharges. Which of the following agents would most effectively treat this child's disorder?

A. Carbamazepine
B. Diazepam
C. Ethosuximide
D. Methylphenidate
E. Phenytoin

In the family shown above, individuals affected with profound deafness are represented by a shaded symbol. The phenotypes of individuals in the fourth generation can best be explained by:

A. autosomal dominant inheritance
B. locus heterogeneity
C. mitochondrial inheritance
D. multifactorial inheritance
E. X-linked dominant inheritance

 

At which point on the diagram above would the length of the myocardial sarcomere be approximately 2 mm with maximal actin-myosin cross bridging?

A. A
B. B
C. C
D. D
E. E

 

Absence of which of the following enzymes would impair the rate-limiting step of glycogenolysis?

A. alpha-1,4-glucan transferase
B. Glycogen phosphorylase
C. Glycogen synthase
D. Phosphoglucomutase
E. UDP-glucose pyrophosphorylase

 

A 34-year-old female is brought to her family physician by her brother. He states that the patient's husband had run away with another woman the previous day. After discovering this, the patient arrived at her brother's home in a "dazed state," was unable to function appropriately the remainder of the evening, and slept fitfully all night. Early in the morning she began to talk to people who were not present, and debated whether or not to kill her children and herself. He denies anything like this ever happening before. Which of the following is the most likely diagnosis?

A. Bipolar disorder
B. Brief psychotic disorder
C. Schizoaffective disorder
D. Schizophrenic disorder
E. Schizophreniform disorder

 

A 34-year-old woman presents with abdominal pain and reports changes in her bowel habits. On pelvic examination a mass is palpated on her right ovary. Laparoscopic surgery is performed and the mass is removed. The gross appearance of the tumor is shown below. What type of tumor is it?





A. Choriocarcinoma
B. Dysgerminoma
C. Fibroma
D. Granulosa-theca cell
E. Teratoma

 

A 55-year-old woman is receiving chemotherapy for non-Hodgkin's lymphoma. Several days after a treatment, she notes that she has blood in her urine. Which of the following antineoplastic drugs is most likely responsible for this side effect?

A. Bleomycin
B. Cisplatin
C. Cyclophosphamide
D. Doxorubicin
E. Plicamycin

 


 

ANSWERS

Question 1: BIO GP

A newborn presents with severe acidosis, vomiting, hypotonia, and neurologic deficits. Serum analysis reveals elevated levels of lactate and alanine. These observations suggest a deficiency in which of the following enzymes?

A. Alanine aminotransferase
B. Glutamate dehydrogenase
C. Lactate dehydrogenase
D. Pyruvate carboxylase
E. Pyruvate dehydrogenase

 The correct answer is: E.

Description:
The correct answer is E. Pyruvate dehydrogenase (PDH) catalyzes the irreversible conversion of pyruvate to acetyl-CoA. If PDH is absent, pyruvate will be used in other pathways instead. Pyruvate will be converted to alanine via alanine aminotransferase (choice A) and to lactate via lactate dehydrogenase (choice C).

Glutamate dehydrogenase (choice B) is involved in oxidative deamination, releasing ammonium ion for urea synthesis. Deficiency of this enzyme would not cause the symptoms described.

Pyruvate carboxylase (choice D) is a gluconeogenic enzyme that catalyzes the conversion of pyruvate to oxaloacetate. Deficiency of this enzyme would not cause the symptoms described.

 


Question 2: BESCI GP

A group of patients with lung cancer is matched to a group of patients without lung cancer.
Their smoking habits over the course of their lives is compared. Based on this information, researchers compute the rate of lung cancer in patients who smoke versus those who never smoked. This is an example of

A. case-control study
B. cohort study
C. cross-sectional study
D. longitudinal study
E. randomized control study

 The correct answer is: A.

Description:
The correct answer is A. Case-control studies are retrospective and are as described in the question stem. Case-control studies allow researchers to compute an odds ratio.

In cohort studies (choice B), subjects are assembled on the basis of some common experience (such as attending medical school) and are then monitored for a specified amount of time at regular intervals (e.g., taking USMLE Steps 1, 2, and 3; see also longitudinal studies below) until they develop the outcome of interest (they become practicing physicians) or the follow-up time ends. The cohort study minimizes many of the biases evident in case-control designs and is the definitive observational clinical study. Cohort studies allow researchers to compute a relative risk.

Cross-sectional studies (choice C) usually have more modest goals than those of case-control and cohort studies. A variable or group of variables is measured in a sample of a larger population to get an idea of the distribution and interrelationships of those variables in that population.

Longitudinal studies (choice D) identify individual subjects and follow them over a given period of time. For example, the study of cholesterol-lowering drugs on cardiovascular events requires that the same subject is observed over a significant period of time (e.g., 10 years).

A randomized controlled trial (choice E) is considered the most rigorous and powerful approach to answering a clinical question in which two treatments, strategies, or therapies are compared or when one therapy is compared to placebo. In this type of study, subjects are assigned treatments on a randomized basis.

 


Question 3: PHARM GP


A pharmacologist is examining a new drug with potential sedative properties. He begins by analyzing the pharmacokinetic properties of the drug. Studies of the drug's rate of elimination yield the above data. Which of the following drugs has similar kinetics to the drug being studied?

A. Carbamazepine
B. Cimetidine
C. Ethanol
D. Ketoconazole
E. Phenobarbital

. The correct answer is: C.

Description:
The correct answer is C. The graph presented is classic for zero-order elimination; the graph is a straight line using standard graphical coordinates. Note that the plasma concentration of the drug diminishes linearly with time. Zero-order elimination means that the rate of elimination is constant and is independent of the drug plasma concentration. Another way of saying this is that a constant amount of drug is cleared per unit time. There are very few drugs that exhibit zero-order elimination; examples include alcohol, phenytoin and aspirin (at high concentrations).

The vast majority of drugs exhibit first-order elimination kinetics. First-order elimination means that a constant fraction of the drug is cleared per unit time. Thus, elimination is proportional to the drug plasma concentration. This can be graphically depicted in several different ways. Using standard graphical coordinates, an exponential decrease in the concentration of the drug is seen. If a semi-logarithmic scale is used, the graph will appear as a straight line, similar to the graph in this question.

 


Question 4: PATH ENDO

A 46-year-old woman presents to her doctor complaining of weakness and fatigue. On physical examination, her physician notices a 10-pound weight gain since her last visit 6 months ago. Her blood pressure is 160/100 mmHg. Blood tests reveal serum Na+ 155 mEq/L, K+ 2.8 mEq/L, and a decreased serum renin. Which of the following is the most likely diagnosis?

A. Cushing's syndrome
B. Diabetes mellitus
C. Pheochromocytoma
D. Primary aldosteronism
E. Secondary aldosteronism

Correct, you answered D.

Description:
The correct answer is D. Primary aldosteronism (
Conn's syndrome) is a condition of hyperaldosteronism originating in the adrenal gland. The causes include an aldosterone-secreting adrenocortical adenoma, hyperplasia of the zona glomerulosa, and very rarely, an adrenal carcinoma. It is characterized by hypertension secondary to sodium retention, hypokalemia, and a decreased serum renin due to a negative feedback of increased blood pressure on renin secretion.

Cushing's syndrome (choice A) is the result of increased glucocorticoid production, particularly cortisol. Physical signs typically include "moon facies," truncal obesity, "buffalo hump," and purple abdominal striae.

Diabetes mellitus (choice B) is a condition of inadequate insulin production that presents with hyperglycemia and ketoacidosis.

Pheochromocytoma (choice C) is a rare tumor of chromaffin cells occurring most commonly in the adrenal medulla. The tumor secretes epinephrine and norepinephrine, resulting in secondary hypertension.

Secondary aldosteronism (choice E) results from an activation of the renin-angiotensin system caused by renal ischemia, edema, and renal tumors. In contrast to primary aldosteronism, secondary aldosteronism is associated with increased serum renin.

 


Question 5: IMMU GP

A 38-year-old woman complains of cold, painful fingertips, as well as difficulty swallowing and indigestion. Physical examination is remarkable for a thickened, shiny epidermis over the entire body, with restriction of movement of the extremities, particularly the fingers, which appear claw-like. Which of the following autoantibodies is likely to be found in this patient's serum?

A. Anti-DNA topoisomerase I (anti-Scl-70)
B. Anti-double-stranded DNA (ds DNA)
C. Anti-IgG
D. Anti-Sm
E. Anti-SS-A

 The correct answer is: A.

Description:
The correct answer is A. This patient is suffering from systemic sclerosis, also called scleroderma. Antibodies to topoisomerase I (anti-Scl-70) occur in up to 70% of patients with diffuse systemic sclerosis, but only rarely in other disorders. Systemic sclerosis is characterized initially by excessive fibrosis and edema of the skin, especially the hands and fingers, producing sclerodactyly (characteristic changes in the fingers, which resemble claws). Raynaud's phenomenon is common. The diffuse type of systemic sclerosis generally spreads to include visceral organs such as the esophagus (producing dysphagia), the lungs (producing pulmonary fibrosis), the heart (leading to heart failure or arrhythmia), and the kidneys (renal failure causes 50% of scleroderma deaths). Females are affected more than males (3:1 ratio). A more restricted variant of systemic sclerosis with a somewhat more benign course is CREST syndrome (Calcinosis, Raynaud's syndrome, Esophageal dysmotility, Sclerodactyly, and Telangiectasia), characterized by the presence of anti-centromere antibodies (although 10% of CREST patients will have anti-topoisomerase antibody as well).

Anti-ds DNA (choice B) is characteristic of systemic lupus erythematosus, but is not common in patients with systemic sclerosis.

Rheumatoid factor is an autoantibody directed against IgG (choice C). It is found in patients with rheumatoid arthritis.

Anti-Sm (Smith antigen; choice D) is also characteristic of SLE rather than systemic sclerosis.

Anti-SS-A (choice E) is typically seen in Sjögren's syndrome (although it may also be seen in SLE).

 

 

  • ANA [ANTINUCLEAR ANTIBODIES] SYSTEMIC LUPUS
  • ANTI dsDNA , ANTI SMITH = SPECIFIC FOR SYSTEMIC LUPUS
  • ANTI HISTONE = DRUG INDUCED LUPUS
  • ANTI IGG =RHEUMATOID FACTOR = RHEUMATOID ARTHRITIS
  • ANTI-NEUTROPHIL= VASCULITIS
  • ANTI-CENTROMERE= SCLERODERMA [CREST]
  • ANTI Scl70= SCLERODERMA SYSTEMIC
  • ANTI-MITOCHONDRIAL = FIRST BILIARY CIRRHOSIS
  • ANTI-GIADIN= CELIAC DISEASE
  • ANTI BASEMENT MEMBRANE = GOODPASTURE SYNDROME
  • ANTI EPITHELIAL CELL = PEMPHIGUS VULGARIS
  • ANTIMICROSOMAL= HASHIMOTO’S THYROIDITIS

 


Question 6: BIO GP

A 26-year-old woman and her 29-year-old husband have been trying to have a child for the last 3 years. During this time the woman has had 5 spontaneous abortions. The karyotypes of the mother, father, and the most recently aborted fetus all contained 46 chromosomes, and all pairs were normal except for the pairs shown below.



The event that led to the child's abnormal karyotype was:

A. adjacent I segregation
B. adjacent II segregation
C. alternate segregation
D. a recombination event within a paracentric inversion
E. a recombination event within a pericentric inversion

You answered D. The correct answer is: B.

Description:
The correct answer is B. The couple is experiencing infertility and a high rate of spontaneous abortions because the father has a reciprocal translocation between chromosome 7 and chromosome 12. His karyotype is 46, XY, t(7;12) (12qter ® 12q23::7p13 ® 7qter; 12pter ® 12q23::7p13 ® 7pter). During meiosis I, the four chromosomes involved in the translocation will pair together in a structure called a quadrivalent, which is pictured below.



There are three ways that the quadrivalent can separate during meiosis I. They are called adjacent I, adjacent II, and alternate segregation. Adjacent II segregation occurs only when the breakpoint of the translocation is so close to the centromere of a chromosome that the cell cannot distinguish between the centromeres. In this specific case, the fetus received a normal chromosome 7 and a normal chromosome 12 from his mother. From his father, the fetus received a normal chromosome 7 and a chromosome composed of 12qter ® 12q23::7p13 ® 7qter. This gives the fetus a total of three copies of the material from 7p13 ® 7qter, and only one copy of material from 12pter ® 12q23. Note that the zygote has three copies of the centromere of chromosome 7, the hallmark of adjacent II segregation. Since the zygote is unbalanced, it aborts.

Adjacent I segregation (choice A) produces cells that contain different centromeres, but have duplication and deletions that lead to spontaneous abortion. In this case, the products of adjacent I segregation would be 12qter ® 12q23::7p13 ® 7qter and the normal chromosome 12, or 12pter ® 12q23::7p13 ® 7pter and the normal chromosome 7.

Alternate segregation (choice C) yields the only outcomes that produce viable progeny. Two products are produced. One contains a normal chromosome 7 and a normal chromosome 12, and the second product contains the balanced translocation. This cell has all the information it needs; it is just arranged in a unique order. In this case, 12qter®12q23::7p13®7qter and 12pter®12q23::7p13®7pter.

A recombination event within a paracentric inversion (choice D) leads to the formation of an acentric fragment and a dicentric bridge. This cell will not complete meiosis.

A recombination event within a pericentric inversion (choice E) leads to chromatids that are duplicated for material on one of the arms and deleted for material on the other arm.

Segregation from a quadrivalent ring results in three different patterns of 2 x 2 segregation.

 

 

 

Translocations in man:

 

1.     Down’s syndrome is caused by trisomy #21, and usually stems from primary or secondary non-disjunction.  Thus,

 

      2n = 46N  vs  2n = 47D

 

2.     About 5% of Down’s individuals are 2n = 46D, and all 2n = 46D individuals have one parent that is 2n = 45N, i.e.,

 

                                                      2n = 46N  vs  2n = 46D  vs  2n = 45N

 

3.     Chromosome studies indicated involvement of chromosomes #15 and #21, and based on observations of “standard” (normal) karyotypes, a non-reciprocal translocation between chromosomes #15 and #21 was hypothesized to be responsible for the 2N = 46D individuals.

 

 

 

a)     One product of the translocation (the 15/21 chromosome) contained most or all of the euchromatin of both chromosomes; whereas the reciprocal product (the 21/15 chromosome) contained mostly heterochromatin and a chromosomal NOR.  Loss of the 21/15 chromosome thus was not expected to have phenotypic consequence.

 

 

 

b)     Matings between a 2n = 45N (translocation heterozygote) and a 2n = 46N normal individual would thus yield…

 

 

 

2/3 aneuploid gametes:  zygotic lethality except for the 2n = 46D

 

1/3 euploid gametes:  normal zygotes, one of which is a translocation heterozygote

 

i)       Note that the tendency to have Down’s children can be inherited.

 

 

 

ii)      Note also that translocations between D group (#13-#15) and G group (#21, #22) chromosomes in humans occur much more frequently than translocations between all other chromosomes in the human complement.

 

 

 

(a)    Because both D and G group chromosomes in the human complement carry NORs, exchanges may arise between “homologous regions” of “non-homologous” chromosomes, and may represent a “price” for the existence of multiple NOR-bearing chromosomes.

 

 


Question 7: ANAT GI

A 70-year-old woman undergoes a gastrectomy for Zollinger-Ellison syndrome. Her doctor informs her that she will need to take intramuscular vitamin B12 shots for the rest of her life. Absence of which of the following cell types is responsible for this vitamin replacement requirement?

A. Chief cells
B. G cells
C. Goblet cells
D. Mucous neck cells
E. Parietal cells

Correct, you answered E.

Description:
The correct answer is E. The parietal cells of the stomach produce intrinsic factor, a glycoprotein that binds vitamin B12 in the lumen of the stomach and facilitates its absorption in the terminal ileum. Patients without a stomach and those with pernicious anemia (autoimmune destruction of parietal cells) will require B12 replacement therapy. Recall that B12 deficiency will lead to megaloblastic anemia and the USMLE-favorite picture of a blood smear with hypersegmented neutrophils. Note that parietal cells also synthesize and secrete HCl.

Chief cells (choice A) are responsible for secreting pepsinogen, the precursor to pepsin.

G cells (choice B) are gastrin-secreting cells
. Gastrin stimulates secretion of acid by the parietal cells found in the body and fundus of the stomach. Zollinger-Ellison syndrome is caused by a pancreatic or duodenal tumor that secretes gastrin (a gastrinoma). It is characterized by the development of severe peptic ulcer disease.

Goblet cells (choice C) are part of the mucosa of the small intestine, not the stomach.
They produce glycoproteins (mucins) that protect and lubricate the lining of the intestine.

Mucous neck cells (choice D) are mucus-secreting cells located in the necks of the gastric glands.

 

Cholecystokinine =I cell of duodenum and jejunum

Secretin = s cell duodenum

Somatostatin = d cells in pancreatic islets, gi mucosa

 

Introduction to Glandular Tissue

Glands are organized arrangements of secretory cells.  All exocrine glands (and also most endocrine glands), are composed of epithelial tissue.

Although most glands give the appearance of being "solid" tissue, their epithelial nature is expressed by the organization of their cells, with each cell attached laterally to its neighbors.  Every exocrine secretory cell has some portion of its plasma membrane exposed to an external surface, communicating with the outside of the body by a system of ducts.

In most glands, the secretory cells are organized into secretory units, which are described according to their shape as tubules, acini, or cords.  

Click here to see an example of a very simple gland, from frog skin.


BASIC TERMINOLOGY

Histologically, glands are described using some standard vocabulary, with which you should be familiar.

Simple / Compound

The simple / compound distinction is based on on duct shape.  

A simple gland has an unbranched duct (or no duct at all).  There is only a single secretory unit (acinus or tubule).  Examples include sweat glands, gastric glands, intestinal crypts, and uterine glands.

A compound gland has a branching duct.  Salivary glands and pancreas are familiar examples.  Compound glands are typically fairly bulky and contain very many individual secretory units (acini or tubules).

Random tissue sections seldom show ducts branching.  Nevertheless, the appearance of multiple duct profiles, in various sizes, provides evidence of a branching duct system.


Acinus / Tubule / Cord

Each secretory unit of a gland consists of cells arranged into an acinus, a tubule, or a cord.  Each of of these arrangements has a different and characteristic appearance when viewed in section.

Acinus (or alveolus)

An acinus (from Latin, grape) is a small ball of secretory epithelial cells containing a tiny central lumen.  Acini are usually formed by serous cells.  [Acini are sometimes called alveoli, from L., small cavity.]

A typical acinar cell is shaped like a pyramid.  Its basal surface, located at periphery of the acinus, rests on the basement membrane separating the acinus from the underlying stroma.  Its lateral surfaces (the sides of the pyramid) are attached to adjacent secretory cells.  Its apical surface is free and faces the acinar lumen, which communicates by duct with the outside.   The acinar cell's cytoplasm is also visibly polarized, usually with basophilic basal cytoplasm and variously-staining secretory granules concentrated in apical cytoplasm.  For more, see serous cells.

A compound acinar gland can be quite accurately modelled as a bunch of grapes embedded in Jello™.  The grapes are the acini, the branching stems are the ducts, and the Jello™ represents the rest of the stroma.  Major and minor branches of the bunch represent lobes and lobules, respectively, separated by greater amounts of connective tissue.

In routine tissue sections, most acini are cut in random planes and look like solid lumps, made of cells having various sizes and shapes.  The lumen of an acinus is typically tiny (i.e., much smaller than a cell) and so is visible only when an acinus is sliced neatly across the middle.  In such a slice, the cells look like slices of pie, with the lumen in the center.

Tubules

Secretory cells may also arrange themselves into secretory tubules (in contrast to the small balls of cells which comprise secretory acini).  Sweat glands are probably the most familiar tubular glands.  Other tubular glands include gastric glands (in the lining of the stomach), uterine glands, and various mucous glands of the GI system.

Because tubules are elongated, random sections commonly include the lumen as well as the secretory cells themselves (in contrast to the situation with acini).  But interpretation of the sectioned appearance of tubular glands will depend on whether the tubules are simple or branched, on whether they are straight or twisted, and on whether or not adjacent tubules lie parallel to one another.

Cords

Cords are arrangements of cells attached to one another to form sheets.  In section, the predominant pattern appears linear, even though the lines may twist and branch.

Cords are a common arrangement for epithelial cells that are specialized for endocrine secretion.  The cells retain an epithelial character, attached to neighboring cells, even though they may no longer comprise a surface barrier between interstitial space and a secretory lumen that leads to the outside.  Examples of endocrine cells arranged into cords include the epithelial cells of pancreatic islets, parathyroid, adrenal cortex, and liver.

The liver (in the thumbnail above) is notable for having cells arranged into cords in spite of its major exocrine function.  In order to maintain communication with ducts, the liver cords contain a network of intercellular channels called bile canaliculi.


Endocrine / Exocrine

The suffix -crine refers to secretion; the prefix endo- or exo- tells where the secretory product goes.

The product of exocrine glands leaves the body proper, either by direct secretion onto the body's surface (e.g., sweat) or into the lumen of an organ (e.g., gastric juice) or else by flowing through a system of ducts (e.g., saliva, pancreatic enzymes, bile).  The cells of exocrine glands are generally arranged into secretory units in the form of acini or tubules (although the liver has a remarkable arrangement of cords).

The product of endocrine glands is secreted into interstitial fluid and hence into capillaries and general circulation.  The cells of endocrine glands are often arranged into cords adjacent to capillaries or sinusoids.

Link to the endocrine system.


Serous / Mucous / Mixed

The serous / mucous distinction is based on the secretory cell's product -- whether it is a clear, watery solution of enzymes (serous, like serum) or else a glycoprotein mixture (mucous, like mucin).  These two categories of secretory products come from two distinct categories of cells, each with a characteristic appearance.

Mixed glands (e.g., most salivary glands) contain both types of cells.  Glands which contain only one of these two cell types may be described either as serous glands (e.g., parotid gland or pancreas) or as mucous glands (e.g., Brunner's glands).  

Serous Secretion

Serous cells are specialized to secrete an enzyme solution. Examples include serous cells of the salivary glands, exocrine cells of the pancreas, gastric chief cells, and Paneth cells of intestinal crypts.  Serous cells of the pancreas and the salivary glands are typically organized into secretory units called acini.

In routine light microscopy, serous cells are distinguished by basophilic basal cytoplasm, a centrally-located nucleus, and variously-staining secretory vesicles (zymogen granules) in apical cytoplasm.  These features are all associated with organized mass production of protein for export.  More.

Mucous Secretion

Cells which are specialized to secrete mucus are called mucous cells.  Examples include secretory cells of the salivary glands, esophageal glands, stomach surface, pyloric glands, and Brunner's glands of the duodenum.  These cells are typically organized into tubular secretory units.

Goblet cells are mucous cells which stand alone within the intestinal epithelium.  Goblet cells take their name from their characteristic shape, with a broad opening at the apical end and a narrow, "pinched" base.  Cells with this goblet shape are also characteristic of the respiratory tract and the female reproductive tract.

In routine light microscopy, mucous cells are most conspicuously distinguished by "empty"-appearing (i.e., poorly stained) apical cytoplasm and by densely-stained, basal nuclei.  More.


Ducts

Ducts are relatively simple tubular structures which are (usually) easily distinguished from blood vessels by their conspicuous cuboidal to columnar epithelial lining.  Blood vessels, in contrast, are lined by simple squamous endothelium.

The glandular cells which comprise ducts generally receive much less attention than those which actually secrete the gland's product.  However, the complete understanding of a gland requires some awareness of and attention to the duct system through which it drains. Ducts are not just passive "plumbing".  Some duct segments actively modify the secretory product passing through, concentrating it by removing water).

For the purpose of describing duct structure and function, especially in compound glands which include branching ducts of various sizes and appearances, some special terminology can be useful.  (By and large, the distinctions that these terms allow represent minor details rather than essential knowledge.)

Intercalated / Striated

Intercalated ducts are small ducts which drain individual secretory units.  These are usually inconspicuous, lined by a simple epithelium consisting of low cuboidal cells.

In some glands, intercalated ducts lead to striated ducts lined by a simple epithelium consisting of conspicuous cuboidal to columnar cells.  In the basal cytoplasm of these cells, fine striations are visible at high magnification.  

The cells of the striated ducts are specialized for concentrating the secretory product that is flowing duct.  They do this by pumping water and ions across the duct epithelium, from the duct lumen and into interstitial fluid.  This striated duct function is carried to extremes in the proximal and distal tubules of the kidney.

Ultrastructurally, striated duct cells display extensive infoldings of the basal membrane.  These folds are closely associated with mitochondria that provide ATP for the membrane pumps.  In light microscopy, the basal folds and mitochondria are sometimes visible as basal striations, hence the name striated duct.

Secretory / Excretory

Both intercalated and striated ducts are sometimes called secretory ducts.  They are located within lobules (intralobular).  More distal ducts (interlobular), sometimes called excretory ducts, are generally passive conducting tubes.  Their size varies, depending on how many branches have converged proximally.  Larger excretory ducts may be lined by stratified cuboidal epithelium.

It is sometimes convenient to refer to ducts by location within the gland.  The following terms are all directly descriptive.  Intra- means within.  Inter- means between.  Lobes and lobules are clusters of secretory units served, respectively, by major and minor branches of the duct tree.  Within a lobule, individual secretory units are separated from one another by little more than basement membranes and capillaries.  In contrast, the stroma which separates lobules and lobes consists of thicker septa of connective tissue.  (The distinction between lobes and lobules is arbitrary; lobes are evident upon gross inspection while lobules are evident to low power microscopy.)

Intralobular -- Located within lobules, with no more connective tissue intervening between ducts and secretory units (i.e., acini or tubules) than between adjacent secretory units.  Intercalated and striated ducts are intralobular.

Interlobular -- Located between lobules, within the thin connective tissue septa that separate lobules.  All interlobular ducts are excretory.

Interlobar -- Located between lobes, within conspicuous, thick connective tissue septa that separate lobes.  All interlobar ducts are excretory.


Parenchyma / Stroma

The parenchyma of an organ consists of those cells which carry out the specific function of the organ and which usually comprise the bulk of the organ.  Stroma is everything else -- connective tissue, blood vessels, nerves, and ducts.  

In most glands, the parenchyma consists of secretory epithelial cells.  However, the parenchyma / stroma distinction can be convenient for describing not only glands but also other organs and even tumors.  Examples:

·         Cardiac muscle cells comprise the parenchyma of the heart.  Everything else is stroma.

·         Nephrons comprise the parenchyma of the kidney.  Everything else is stroma.

·         Hepatocytes comprise the parenchyma of the liver.  Everything else is stroma.

·         Neurons comprise the parenchyma of the brain.  Everything else is stroma.

·         Cancer cells comprise the parenchyma of malignant neoplasms.  Everything else is stroma.

Because parenchyma often seems more interesting, stroma is commonly ignored as just boring background tissue.  But no organ can function without the mechanical and nutritional support provided by the stroma.  In any gland, connective tissue and capillaries of the stroma envelope every acinus, tubule, or cord, although they are often inconspicuous.  

Pay attention to the stroma.  If an organ is inflamed, the signs of inflammation appear first in the stroma.

Specialized Cells of the GI System

The GI system includes a number of highly specialized cell types, each differentiated to perform a specific function.  

Listing by region

Listing by cell name.

Listing by function.

Most of the listed cells are epithelial, since variously specialized epithelia carry out most of the functions which are specific to the GI system.  Other tissues are also vitally important to GI function, but are basically similar in all organ systems and are not individually listed here.  See separate pages for smooth muscle, for cells of connective tissue (including immune system), and for nerve cells.


Listing of Cells by Region


Listing of Cells by Function

Intestinal
Absorption

Absorptive Cells (Enterocytes)

Mucus
Secretion

Mucous Cells
(includes salivary, esophageal, pyloric, and Brunner's glands)

 

Goblet Cells

Gastric Surface
Mucous Cells


Listing of Cells by Name

Absorptive Cells
(Enterocytes)

Goblet Cells

Paneth Cells

Gastric Chief Cells

Gastric Parietal Cells

Gastric Surface
Mucous Cells

Serous Cells

Mucous Cells

Pancreatic
Acinar Cells

Pancreatic
Islet Cells

Enteroendocrine Cells

Stem Cells

Hepatocytes

Kupffer Cells

Fenestrated Hepatic
Endothelial Cells

Taste Buds

Smooth Muscle Cells

Myoepithelial Cells


Intestinal Absorptive Cells

Absorptive cells, or enterocytes, are the predominant cell type in the epithelium of the small intestine and colon.  These cells are specialized for absorption of nutrients across the apical plasma membrane and export of these same nutrients across the basal plasma membrane.  Upon release at the basal end of the cell, nutrient molecules diffuse into connective tissue space and eventually into capillaries or lacteals of the lamina propria.

The apical surface area of each absorptive cell is greatly increased by evagination into a dense array of microvilli, visible microscopically as the brush border.

In the small intestine, brush border enzymes contribute to digestive breakdown (these enzymes are lacking from absorptive cells of the colon).

The microvilli are supported by an underlying meshwork of microfilaments which comprise the terminal web.  At the lateral edges of each cell, the the terminal web reinforces the junctional complex that attaches adjoining cells.  (This site is sometimes visible microscopically as a terminal bar at the apical corners of each cell.).

A pale (poorly stained) region above the nucleus indicates the location of the Golgi apparatus, where some absorbed nutrients are processed for transport.

Associated cell types:  Interspersed among the many absorptive cells of the intestinal epithelium are scattered goblet cells, occasional enteroendocrine cells (which are difficult to distinguish in routine preparations), and occasional wandering cells of the immune system (e.g., lymphocytes, eosinophils).  

The life-span of absorptive cells is short, only a few days.  During this time, the cells migrate from deep in crypts, where they are formed by dividing stem cells, to the surface epithelium where they eventually undergo apoptosis.  Along villi, this migration is sometimes called "the epithelial escalatory".  Look for apoptosis at the tips of villi.

Consult your histology textbook and/or atlas for additional detail and electron micrographs of these cells.


Goblet cells

Goblet cells are scattered among the absorptive cells in the epithelium of the small intestine and colon.  These cells are specialized for secretion of mucus, which facilitates passage of material through the bowel.  The name "goblet" refers to the cell's shape, narrow at the base and bulging apically.  

(Similar cells may also be found in the respiratory and reproductive tracts.)  

The apical end of each goblet cell is occupied by a large mass of mucus, which compresses adjacent cells (thus conferring the characteristic "goblet" shape) and displaces the nucleus toward the basal end of the cell.  As in other mucous cells, the nucleus is compact and intensely-stained.

Goblet cells comprise an integral part of the epithelium, attached by junctional complexes (evidenced in light microscopy as the "terminal bar") to adjacent absorptive cells.

The proportion of goblet cells to absorptive cells increases along the entire length of the bowel, with relatively few in the duodenum and very many in the colon.  The colonic epithelium gives the superficial impression of being almost entirely goblet cells, but there are still several times as many absorptive cells.  The goblets are just more conspicuous, with their bulging mucus droplets.

Consult your histology textbook and/or atlas for additional detail and electron micrographs of these cells.


Mucous cells

Cells which are specialized to secrete mucus are called mucous cells.  In routine light microscopy, mucous cells are most conspicuously distinguished by their "empty" appearance (i.e., poorly stained cytoplasm) and densely-stained, basal nuclei.

The nucleus in a typical mucous cell gives the impression of having been displaced and compressed by the mass of mucus accumulated in the apical end of the cell.

Mucus does not stain well with standard acidic or basic dyes, but is demonstrated with the Periodic Acid Shiff procedure (PAS stain).

Examples of mucous cells from the GI may be found in salivary glands, esophageal glands, stomach surface, and Brunner's glands of the duodenum.  Goblet cells, which stand alone within a surface epithelium, are a characteristic feature of the intestine (as well as the respiratory tract and the female reproductive tract).

Consult your histology textbook and/or atlas for additional detail and electron micrographs of these cells.


Serous Cells

Serous cells, exemplified by pancreatic acinar cells, are specialized for exocrine enzyme secretion.

A serous cell's cytoplasm is visibly polarized, with basophilic basal cytoplasm and variously-staining secretory vesicles (zymogen granules) concentrated in apical cytoplasm.  The basal basophilia is due to a concentration of protein synthetic organelles (ribosomes on rough endoplasmic reticulum).  The Golgi apparatus is usually located midway along the cell, typically in a supranuclear position.  

Examples of serous cells include acinar cells of pancreas and salivary glands, gastric chief cells, and intestinal Paneth cells.  

Consult your histology textbook and/or atlas for additional detail and electron micrographs of these cells.


Paneth cells

Paneth cells are secretory cells located at the ends of intestinal crypts.  The function for these cells is secretion of anti-bacterial proteins into the crypt lumen, thereby providing protection for the stem cells which line the crypt walls.

Paneth cells have typical serous-secretory appearance, with basophilic basal cytoplasm (containing protein-synthetic rough endoplasmic reticulum) and apical secretory vesicles (zymogen granules).  

The secretory vacuoles of Paneth cells contain lysosomal enzymes, with anti-bacterial function.  

Consult your histology textbook and/or atlas for additional detail and electron micrographs of these cells.


Myoepithelial cells

Myoepithelial cells are contractile epithelial cells which forcibly express the contents of a gland.  Although myoepithelial cells function like smooth muscle, they are typically located within a glandular epithelium, between the secretory cells and the basement membrane.  Each myoepithelial cell has long cytoplasmic processes which wrap around a secretory unit.  Hence, contraction of the myoepithelial processes can squeeze secretory product from the secretory unit into its duct.

Myoepithelial cells contribute significantly to the secretory activity of sweat glands, salivary glands, and mammary glands.

In routine histological preparations, myoepithelial cells are inconspicuous.  The cell body can sometimes be noticed as an "extra" nucleus beneath some secretory cells.  The contractile process are thin eosinophilic strands.  In routine H&E stained sections of sweat glands, these can often be seen as small pink triangles underlying the secretory cells.  In sweat glands and mammary glands, myoepithelial processes are normally difficult to observe without special stains.

·         For a nice image of myoepithelial cells (in breast) stained with immunoperoxidase, see WebPath.

Consult your histology textbook and/or atlas for additional detail and micrographs of these cells


Taste Cells

The sensation of taste is mediated by elongated sensory cells which occur in clusters called taste buds on fungiform and circumvallate papillae of the tongue.  Taste buds interrupt and extend across the tongue's stratified squamous epithelium.

These sensory cells are normally replaced every 10-14 days by division and differentiation of stem cells, which is handy since they are exposed and easily damaged (e.g., by a bite of too-hot pizza).

Associated cell types:  In addition to sensory cells, the elongated cells of the taste bud include support cells, which are sometimes (rather pretentiously) called sustentacular cells (from the same root as "sustain").  Stem cells are normally visible in taste buds as shorter cells with round nuclei which appear at the basal end of the tastebud.

Consult your histology textbook and/or atlas for additional detail and electron micrographs of these cells


Hepatocytes

Although all cells are general-purpose metabolic factories, most mature cell types are functionally specialized to play one particular role in the economy of the larger body.  In contrast, hepatocytes of the liver are remarkable for the breadth of their "specialization".  Among other functions, hepatocytes:

·         form and secrete bile

·         store glycogen and buffer blood glucose

·         synthesize urea

·         metabolize cholesterol and fat

·         synthesize plasma proteins

·         detoxify many drugs and other poisons

·         process several steroid hormones and vitamin D

The cellular structure of hepatocytes is correspondingly rich.  Instead of displaying a concentration of one particular organelle, hepatocytes have lots of everything -- lots of mitochondria, lots of ribosomes, lots of Golgi bodies, lots of endoplasmic reticulum (both rough and smooth), lots of stored glycogen, lots of lysosomes, lots of plasma membrane (with microvilli on the free surface).  They even have lots of nucleus (yes, hepatocytes may be polyploid and commonly have two full-size nuclei).

When an introductory biology book illustrates "the cell", the representative example is often a hepatocyte -- because hepatocytes offer splendid and numerous examples of most cellular organelles.

In appearance, hepatocytes are boxy (cuboidal) cells with one or two large euchromatic nuclei and with abundant, grainy cytoplasm that stains well with both acid and basic dyes (reflecting the abundance of various cellular constituents).  Because individual liver cells have an indefinite lifespan, they may accumulate abundant lipofuscin (yellow-brown "wear-and-tear" pigment), especially with advancing age.

Hepatocytes are arranged into cords, in which each hepatocyte is attached to its neighbors in a two-dimensional sheet.  On either side of the cord, each hepatocyte faces the space of Disse, across which it communicates freely with adjacent sinusoids.

Hepatocytes are epithelial, but their epithelial nature is expressed in a rather peculiar way.

An ordinary epithelial cell sits on its basal surface, has an apical surface exposed to the external space, and is attached to its neighbors along its lateral surface.  In principle, hepatocytes follow this same plan -- but they have a unique topology.  

A typical hepatocyte has two basal surfaces, on opposite ends of the cell where it faces the sinusoids on either side of the cord in which it resides.  

The apical surface of a hepatocyte occurs along a band around the cell's middle, half-way between the opposing basal surfaces.  It is across this surface that bile is secreted. The edges of this apical surface are attached by junctional complexes to those of adjacent hepatocytes, thereby forming the bile canaliculi.  These bile canaliculi form a network encircling each hepatocyte and sealed within the hepatic cord.  Along this network (which is shaped like chicken-wire), bile can seep toward the periphery of the lobule and hence into the proper bile ducts which are found only portal areas.

(Bile canaliculi are barely visible in routine microscopic preparations, at sites where the boundary between adjacent hepatocytes has been cut neatly and perpendicularly.  They can be clearly demonstrated with special stains.)

The lateral surfaces of a hepatocyte take the form of two broad bands which wrap around the cell between the two basal surfaces, separated by the narrow band of apical surface.  These lateral surfaces attach the cell to its neighbors within the cord, with junctional complexes sealing off and separating the bile canaliculi (at the apical surface) from the plasma-containing space of Disse (at the basal surfaces).

Consult your histology textbook and/or atlas for additional detail and electron micrographs of these cells.


Endothelial Cells

Endothelial cells are simple squamous cells which line the entire vascular system (including lymphatic channels).  Endothelial cytoplasm is inconspicuous in routine light microscopy.  Typically only the nuclei are visible, at the boundary between the lumen and the wall of a vessel.  

Endothelial nuclei typically appear thin and dark, in cross section.  However, occasionally endothelium lies parallel to the plane of section.  In this case the nuclei may appear very large, round, and pale.

Although endothelial cells appear rather uninteresting under the microscope, these are important cells.  They are situated at a critical location, between the blood and all other body cells.  They secrete substances which control local blood flow and blood coagulation, and they are active participants in white blood cell emigration during inflammation.

Continuous endothelium.  Throughout much of the body, the capillary endothelial lining is continuous, with neither large gaps between cells nor holes through cells.  Materials pass through the endothelium either by diffusion or via rapid vesicular transcytosis.  (In most of the brain, a lack of transcytotic vesicles accounts for the blood brain barrier -- the only substances which cross such a barrier are those which can diffuse through plasma membranes or those for which specific membrane channels exist.)  

Fenestrated endothelium.  In a few special locations -- notably in the sinusoids of the liver, in the glomeruli of the kidney, and in most endocrine glands -- the endothelium is fenestrated (i.e., full of holes -- from fenestra, window).  In the liver, where there is also no basement membrane, the fenestrations permit blood plasma to wash freely over the exposed surfaces of the hepatocytes through the space of Disse.

Consult your histology textbook and/or atlas for additional detail and electron micrographs of these cells.


Kupffer Cells (Liver Macrophages)

Kupffer cells are macrophages found in the sinusoids of the liver.  These cells have standard macrophage / monocyte specialization (i.e, receptor-mediated phagocytosis, lysosomal digestion).  This particular population of macrophages are especially significant, with responsibility not only for cleaning bacteria out of the portal blood stream (the "dirty" blood" from the intestine), but also for removing worn-out red blood cells and recycling hemoglobin (a job shared with macrophages of the spleen).

Kupffer cells are closely associated with the endothelial lining of the liver.  Lying along side or draped across the liver sinusoids, the Kupffer cells are not easily distinguished from the endothelial cells.

Historical note:  The term reticuloendothelial system refers to the macrophages of the liver, spleen and lymph nodes, i.e., those organs with elaborate endothelially-lined channels supported by reticular connective tissue.  The name reflects former confusion about the distinction between endothelial cells and the scattered population of macrophages (monocytes, histiocytes).  Macrophages can be readily labelled experimentally through their phagocytosis of injected carbon particles.  However, endothelial cells are also labelled by the same procedure.  Although endothelial cells are not dramatically phagocytotic, they do shuttle some materials across the endothelial lining via small endocytotic and exocytotic vesicles.

Consult your histology textbook and/or atlas for additional detail and electron micrographs of these cells.


Gastric Surface Mucous Cells

The protective cells which line the surface of the stomach (including gastric pits) are called surface mucous cells.  These cells are critical for resisting attack by digestive acid and enzymes.  

Any disruption of these cells' function can lead to an ulcer.  See WebPath (low mag), WebPath (high mag), or Milikowski & Berman's Color Atlas of Basic Histopathology, pp. 240-241.

The appearance of surface mucous cells is rather different from that other mucous cells

. Their nuclei are not compressed basally (at least, not usually as much so as other mucous cells) and their apical mucus droplets show some affinity for eosin (i.e., pink-staining in H&E).  These differences presumably reflect the fact that the mucus secreted by these cells has a special composition to resist digestion.

Individual surface mucous cells also differ in shape from intestinal goblet cells.  Since the mucosal surface of the stomach consists of these cells and no others, individual surface mucous cells cannot bulge apically like the goblet cells.

Consult your histology textbook and/or atlas for additional detail and electron micrographs of these cells.

 


 

Gastric Parietal Cells

Parietal cells (oxyntic cells) of the stomach secrete acid, by pumping hydrogen ions across the cell membrane.  These are among the most dramatically differentiated epithelial cells in the body, with functional specialization reflected in their microscopic appearance.  

Parietal cells are relatively large cells, with one or two oval, centrally located euchromatic nuclei.  The cytoplasm is strongly acidophilic, is somewhat grainy in appearance, and is typically less-intensely stained in a zone midway between the nucleus and the cell membrane. This cytoplasmic appearance is closely associated with the parietal cell's functional specialization for secreting acid.  

[Note that cytoplasmic acidophilia or basophilia in histological preparations is no reflection of cytoplasmic pH, but only of the incidental post-fixation staining characteristics of cytoplasmic organelles.  For example, mitochondria happen to be acidophilic.  Ribosomes happen to be basophilic.]

Pumping hydrogen ions against a tremendous concentration gradient (from pH = 7.0 to pH < 1.0) requires energy, which in turn requires mitochondria to produce ATP.  [Consult your physiology references for additional detail about acid production by parietal cells.]  Parietal cells contain so many mitochondria that the cytoplasm appears to be packed with these organelles, leading to the cells' characteristic acidophilia (mitochondria are acidophilic).  

The area of membrane surface across which ions may be pumped is substantially increased by a deep invagination into the cytoplasm (the so-called intracellular canaliculus).  The surface area of this pocket is further increased by microvilli.  Since the lumen of the canaliculus is extracellular, it contains no organelles; therefore, the cytoplasm stains less intensely over the canaliculus.

Parietal cells may be found at any level in the fundic glands, but they are most common in the middle region.

Consult your histology textbook and/or atlas for additional detail and electron micrographs of these cells.


 

 

Gastric Chief Cells

Chief cells of the stomach secrete the digestive enzymes (pepsins) of the stomach and have typical serous-secretory appearance.  

Secretory vesicles (containing pepsinogen) are clearly visible in the apical cytoplasm of chief cells.  Basal cytoplasm is strongly basophilic (especially by comparison with parietal cells), due to the presence of the extensive rough endoplasmic reticulum that synthesizes protein for secretion.

[Note that cytoplasmic acidophilia or basophilia in histological preparations is no reflection of cytoplasmic pH, but only of the incidental post-fixation staining characteristics of cytoplasmic organelles.  For example, mitochondria happen to be acidophilic.  Ribosomes happen to be basophilic.]

Chief cells may be found at any level in the fundic glands, but they are most common in the deeper region, toward the muscularis mucosae.

Consult your histology textbook and/or atlas for additional detail and electron micrographs of these cells.


 

 

Gastric Mucous Neck Cells

Mucous neck cells of the stomach are inconspicuous cells with a typical mucous-secretory appearance.  These cells are most common in the upper ("neck") region of the fundic glands (i.e., near to the glands' openings into the bottoms of gastric pits).  Their specific function remains unclear.

Mucous neck cells are difficult to identify in routine sections of stomach mucosa. When the cells are neatly cut from top to bottom, they can be recognized by the basally compressed (often wedge-shaped) nucleus and the stored mucus that fills the apical cytoplasm.  But in other random planes of section, the poorly-stained mucus is inconspicuous and the nucleus can resemble any of the small dense nuclei found in lamina propria.

Consult your histology textbook and/or atlas for additional detail and electron micrographs of these cells.


 

 

Endocrine Cells
Link to the endocrine system.

Many of the body's cells secrete substances which influence other cells, either locally or at some distance.  Cells generally regarded as endocrine cells are those which are conspicuously specialized for this function.  The gastrointestinal system has two classes of such cells, the islets of Langerhans in the pancreas and the enteroendocrine cells in the mucosal epithelium of the stomach and intestine.

Pancreatic islets are covered elsewhere.  

Enteroendocrine cells comprise a class of columnar epithelial cells, scattered individually among the absorptive cells and exocrine cells that line the mucosa of the GI tract.  They are located most commonly in the necks and deeper in gastric glands and in the lower portions of intestinal crypts.  Each type of enteroendocrine cell is specialized to secrete a particular hormone that influences gastrointestinal secretion or motility.  Consult your physiology text for functional details.

Enteroendocrine cells have traditionally been recognized by their affinity for certain metal stains, hence the older terms chromaffin cells (having an affinity for chromium) and argentaffin cells or argyrophil cells (having an affinity for silver).  With sufficient resolution, these cells can sometimes be recognized in routine light microscopic preparations by their relatively pale cytoplasm with a broad base and a basal concentration of secretory vesicles (in contrast to the apical concentration of secretory vesicles for exocrine serous cells or mucous cells).  Immunocytochemical methods are preferred for demonstrating and properly identifying the various types of enteroendocrine cells.

Consult your histology textbook and/or atlas for additional detail and electron micrographs of these cells.


 

 

Stem Cells

Stem cells are undifferentiated cells which remain capable of cell division to replace cells which die.  In the gastrointestinal tract, epithelial stem cells are essential to continually replenish the surface epithelium.

Microscopically, stem cells lack distinguishing characteristics.  However, their activity can be detected by the presence of mitotic figures, the intensely basophilic masses of condensed chromation which characterizes cells undergoing mitosis.

(Other organ systems also contain stem cells, most notably the hemopoietic cells of bone marrow.  These other regions will not be covered here.)

Stem cells of the gastric mucosa are located at the at the top of the glands where they open into the pits.  These cells are responsible for replenishing the secretory cells of the gastric glands and also the surface mucous cells that protect the stomach surface.  These cells are difficult to notice and even more difficult to identify in routine histological preparations.

Stem cells of the intestinal mucosa line the walls of the crypts and continually replenish the intestinal epithelium, completely replacing all the absorptive cells and goblet cells approximate once every four days.  These cells are inconspicuous when resting, but mitotic figures (intensely basophilic condensed chromatin) are common and easily noticed in the crypts of the small intestine.

The importance of stem cells in the intestinal crypts is illustrated by recovery from cholera.  The cholera toxin kills the intestinal epithelium, leading to loss of bodily fluid across the mucosa, copious diarrhea, massive dehydration, and death within a few days.  However, if patients can be kept hydrated for those few days, epithelial replacement by stem cell division will restore normal function.

Stem cells of the oral cavity and the esophagus, like those of the epidermis, are located in the basal cell layer of the stratified squamous epithelium.

Consult your histology or cell biology textbook for additional information about stem cells.


 

 

Question 8:  GP

A patient presents with right lower quadrant pain, fever, and diarrhea. Physical examination reveals diffuse abdominal tenderness; laboratory examination reveals a moderate leukocytosis, leading to a presumptive diagnosis of acute appendicitis. Surgical exploration of the abdomen reveals mesenteric adenitis, but the appendix is normal. Which of the following organisms is most likely responsible for these signs and symptoms?

A. Clostridium difficile
B. Enterohemorrhagic E. coli
C. Enteroinvasive E. coli
D. Enteropathogenic E. coli
E. Yersinia enterocolitica

 The correct answer is: E.

Description:
The correct answer is E. Many patients with Yersinia enterocolitica infection present with symptoms suggesting appendicitis (although constipation, rather than diarrhea, is common in appendicitis). Surgical exploration of the abdomen reveals mesenteric adenitis (involvement of the lymph nodes located in the mesentery), along with a normal appendix. Mesenteric adenitis is the tip-off that the patient has a Yersinia enterocolitica infection, and culture of the involved lymph nodes or of the stool are generally diagnostic.Following antibiotic use (e.g., clindamycin, ampicillin, or a third-generation cephalosporin), Clostridium difficile (choice A) may cause bloody diarrhea due to pseudomembranous colitis.

Y. enterocolitica

Most important as cause of self-limiting gastroenteritis or enterocolitis

Particularly common among children, outbreaks in day-care centers/schools

Symptoms mild to severe

- mild: diarrhea, abdominal pain

- to severe: fever, severe abdominal pain, mistaken for appendicitis

Occasional Y. enterocoliticaGI infection followed by arthritis of peripheral joints

- 2 - 6 weeks after intestinal infection clears

- called reactive arthritis (Reither's syndrome)

- thought to be caused by T-cells and/or antibodies

- elicited by antigens that cross-react with host antigens, not by infection of the joints

- common in people with histocompatibility antigen HLA-B27 (exposed on surface of host cells)

- antibodies vs. bacterial surface antigens cross-react with human B27, activate C'

- T-cells recognizing B27 may attack host cells

Relatively small number of strains cause disease
Enterohemorrhagic Escherichia coli (choice B) is the causative organism to suspect in a patient who develops severe, hemorrhagic, traveler's diarrhea with bleeding manifestations, hematuria, oliguria, and a microangiopathic hemolytic anemia. This bacteria produces verocytotoxin, which is responsible for the symptoms.

Enteroinvasive E. coli (EIEC; choice C), causes fever, pain, diarrhea, and dysentery, usually following ingestion of contaminated cheese or water.

Enteropathogenic E. coli (EPEC; choice D), causes a watery diarrhea in infants and toddlers secondary to ingestion of contaminated food or water.

Enterotoxigenic E. coli (ETEC; choice E) causes traveler's diarrhea, a watery diarrhea due to ingestion of contaminated food or water. (Mnemonic: Traveler's diarrhea = ETEC = Toxigenic E. coli).


 

 

Question 9: PHARM CV


Mean arterial blood pressure (MAP) and heart rate (HR) measurements were recorded during the intravenous administration of two different drugs. Select the most likely drugs given at the indicated points.

A. X, Acetylcholine Y,phentolamine
B. X, epinephrine Y,hexamethonium
C. X, isoproterenol Y, propranolol
D. X, metaproterenol Y, propranolol
E. X, norepinephrine Y, hexamethonium

 The correct answer is: E.

Description:
The correct answer is E. There are several strategies that are useful in solving drug tracing problems. First, always look at blood pressure first and heart rate second. Blood pressure changes will be the direct result of the administered drug; heart rate changes may be a direct effect or they may result from a baroreceptor reflex. Second, assume that when an agonist effect is gone, so is the agonist; however, assume that an antagonist stays on board for the entire trace. Third, begin by eliminating as many options as possible by examining the initial agonist effect.

The only two agonists that would be expected to raise blood pressure to this degree are norepinephrine (alpha-1, alpha-2, and beta-1 agonist) and phenylephrine (alpha-1 agonist). Thus, choices A, B, C, and D can be eliminated immediately.

Drug Y lowers blood pressure and raises heart rate. Hexamethonium (ganglionic blocker) would lower blood pressure and increase heart rate by blocking the predominant tone of the arterioles (sympathetic) and the heart (parasympathetic). Phentolamine (alpha antagonist) would lower blood pressure by blocking arteriolar alpha-1 receptors. The increase in heart rate would be a baroreceptor reflex.

The second administration of Drug X causes an increase in both blood pressure and heart rate. Only the combination of norepinephrine (NE) and hexamethonium could do this. NE would still increase blood pressure by stimulating end organ receptors; however, the baroreceptor reflex would be blocked by hexamethonium. NE\'s ability to increase heart rate by stimulating cardiac beta-1 receptors is now revealed.

(Choice A) Acetylcholine would cause a decrease in blood pressure by stimulating non-innervated muscarinic receptors present on arterioles. Heart rate would increase because of a baroreceptor reflex. Phentolamine would cause a decrease in blood pressure and an increase in heart rate (baroreceptor reflex). It should have no effect on a subsequent ACh administration.

(Choice B) Epinephrine would cause a small increase in mean blood pressure, but not enough to elicit a baroreceptor reflex. So, an increase in heart rate would be seen as a result of beta-1 stimulation of the heart. Hexamethonium would cause a decrease in blood pressure and an increase in heart rate. Hexamethonium should not affect a subsequent epinephrine administration because a baroreceptor reflex was not initially produced.

(Choice C) Isoproterenol (beta-1 and beta-2 agonist) would cause a decrease in blood pressure (by stimulating arteriolar beta-2 receptors) and an increase in heart rate (a combination of cardiac beta-1 receptor stimulation and baroreceptor reflex). Propranolol (beta-1 and beta-2 antagonist) would decrease blood pressure and heart rate. It would completely block a subsequent administration of isoproterenol.

(Choice D) Metaproterenol (beta-2 agonist) would decrease blood pressure (by stimulating arteriolar beta-2 receptors) and increase heart rate (baroreceptor reflex). Propranolol (beta-1 and beta-2 antagonist) would decrease blood pressure and heart rate. It would completely block a subsequent administration of metaproterenol.

(Choice F) Phenylephrine would increase blood pressure and decrease heart rate (baroreceptor reflex). Phentolamine would decrease blood pressure and increase heart rate (baroreceptor reflex). Phentolamine would completely block a subsequent administration of phenylephrine.


 

 

Question 10: PHYSIO MS

A 21-year-old man is competing in a weight-lifting competition. He lifts 325 lbs over his head and holds it there for 5 seconds. Suddenly, his arms give way and he drops the weights to the floor. Which of the following receptors is responsible for this sudden muscle relaxation?

A. Free nerve ending
B. Golgi tendon organ
C. Merkel's disk
D. Muscle spindle
E. Pacinian corpuscle

Correct, you answered B.

Description:
The correct answer is B. Normally, stretching of muscle results in a reflex contraction: the harder the stretch, the stronger the contraction. At a certain point, when the tension becomes too great, the contracting muscle suddenly relaxes. The reflex that underlies this sudden muscle relaxation is called the Golgi tendon organ reflex, also known as the inverse stretch reflex or autogenic inhibition. The Golgi tendon organ (GTO) is an extensive arborization of nerve endings (encapsulated by a connective tissue sheath and located near the muscle attachment) that is connected in series with the extrafusal skeletal muscle fibers. As a result, GTOs respond to muscle tension rather than muscle length. Increased tension leads to stimulation of Ib afferents, which inhibit the homonymous muscle via spinal interneurons.

Free nerve endings (choice A) are unmyelinated, unencapsulated nerve endings that penetrate the epidermis. These types of receptors respond to pain and temperature.

Merkel's disks (choice C) are comprised of specialized tactile epidermal cells and their associated nerve endings. They are located in the basal layer of the epithelium and are slowly adapting receptors that respond to touch and pressure.

Muscle spindles (choice D) are spindle-shaped bundles of muscle fibers (intrafusal fibers) that are encapsulated by connective tissue. Muscle spindles are arranged in parallel with extrafusal skeletal muscle fibers, so they sense the length of the muscle. They are innervated by Group Ia and II sensory afferent neurons.

Pacinian corpuscles (choice E) are unmyelinated nerve endings surrounded by thin, concentric layers of epithelioid fibroblasts. In transverse section, this receptor resembles a sliced onion. They are found primarily in the deep layer of the dermis, loose connective tissue, male and female genitalia, mesentery, and visceral ligaments. They are rapidly adapting receptors that respond to touch and pressure.


 

 

Question 11: PHYSIO RENAL

A normal volunteer consents to an intravenous infusion of p-aminohippuric acid (PAH). After a short time, the plasma PAH is 0.02 mg/ml, the concentration of PAH in urine is 13 mg/ml and the urine flow is 1.0 ml/min. What is the effective renal plasma flow?

A. 0.26 ml/min
B. 26 ml/min
C. 65 ml/min
D. 260 ml/min
E. 650 ml/min

 The correct answer is: E.

Description:
The correct answer is E. Approximately 90% of a small dose of PAH is cleared by the kidney in a single pass. If it were 100%, then the amount of PAH in urine (concentration ´ urine flow rate) divided by the amount of PAH in plasma would exactly equal the renal plasma flow. Because the extraction ratio (arterial-venous PAH concentration divided by arterial concentration) is 0.9 (90%) instead of 100%, physiologists speak of the quantity UPAHV/PPAH as the effective renal plasma flow (ERPF). So, in this patient, we have (13 mg/ml ´ 1.0 ml/min)/0.02 mg/ml = 650 ml/min.

0.26 ml/min (choice A) can be obtained by multiplying 13 by 0.02 and dividing by 1. This value is far too low to be a normal ERPF, which is typically around 625 ml/min.

26 ml/min (choice B) is 100 times (13 ´ 0.02)/1. This value is far too low to be a normal ERPF, which is typically around 625 ml/min.

65 ml/min (choice C) might indicate that you set the ratio up correctly, but dropped a power of 10 in your calculations. This value is far too low to be a normal ERPF, which is typically around 625 ml/min.

260 ml/min (choice D) is 1000 times (13 ´ 0.02)/1, and is still too low to be a normal ERPF.

Timesaving note: If you remembered that the ERPF is approximately 625 ml/min, you do not really need to calculate anything in this question. Choice E is the only reasonable answer.


 

 

Question 12: ANAT RESP

A 4-day-old male is brought to the pediatric clinic because of breathing difficulties and poor feeding. He coughs, chokes, and spits up milk very soon after beginning to suckle. Physical exam and radiographs reveal the presence of the most common type of tracheoesophageal fistula. The baby's defect likely resulted from:

A. failure of the buccopharyngeal membrane to rupture
B. failure of the tracheoesophageal ridges to fuse
C. incomplete formation of the septum secundum
D. incomplete recanalization of the larynx
E. patent thyroglossal duct

Correct, you answered B.

Description:
The correct answer is B. The tracheoesophageal ridges are two longitudinal ridges that separate the respiratory diverticulum from the foregut. Eventually, they fuse to form a septum separating the esophagus (dorsal) from the trachea (ventral) and lung buds, maintaining a communication only rostrally at the pharynx. Incomplete formation of the tracheoesophageal septum (by fusion of ridges) results in the most common type of tracheoesophageal fistula, whereby the proximal part of the esophagus ends as a blind sac (esophageal atresia), while the distal part is connected to the trachea by a narrow canal just above the bifurcation. This defect occurs in approximately 1 of every 2500 births.

The buccopharyngeal membrane is a bilaminar membrane (ectoderm externally, endoderm internally) separating the stomodeum (mouth) from the pharynx. The membrane ruptures at about 4 weeks. The buccopharyngeal membrane is not involved in formation of the esophagus and trachea and so failure to rupture (choice A) would not lead to tracheoesophageal fistula.

The septum secundum is a membrane that forms on the right side of the developing interatrial wall of the heart. It is not associated with formation of the esophagus and trachea and so failure to close would not lead to tracheoesophageal fistula. Failure of formation of the septum secundum (choice C) leads to a patent foramen ovale, a relatively common atrial septal defect.

Incomplete recanalization of the larynx (choice D) is relatively rare and results in a membrane (laryngeal web) that may partially obstruct the airway. Though there may be difficulty breathing, there should be little problem with swallowing and keeping milk down.

The thyroid gland forms from a primordium associated with development of the tongue that eventually descends into the neck. For a short time, it remains connected to the tongue by a narrow canal called the thyroglossal duct. The duct normally closes, but occasionally, it remains patent (choice E) or develops cysts, which are usually asymptomatic unless they become infected. They do not interfere with breathing or feeding and do not involve the trachea or esophagus.


 

 

Question 13: BIO GP

A 4-month-old blue-eyed, fair-skinned child who appeared normal at birth is brought to the pediatrician because of the development of scaly skin lesions and seizures. On physical examination, the infant appears to be mentally retarded, and a musty odor is noted. Which of the following defects is responsible for this child's illness?

A. Deficiency of alpha-ketoacid decarboxylase
B. Deficiency of hexosaminidase A
C. Deficiency of homogentisate oxidase
D. Deficiency of hypoxanthine-guanine phosphoribosyltransferase
E. Deficiency of phenylalanine hydroxylase

Correct, you answered E.

Description:
The correct answer is E. Phenylketonuria occurs in approximately 1:10000 births. Seemingly normal at birth, these infants later develop seizures, eczematous lesions, and mental retardation. The musty odor is extremely characteristic; if a child is described on the USMLE as smelling musty or having musty-smelling urine, he/she will almost certainly have PKU. PKU results from a defect in phenylalanine hydroxylase, the enzyme responsible for converting phenylalanine to tyrosine. Without this enzyme, phenylalanine is converted to phenylpyruvate, a phenylketone that spills into the urine. (If you did not recall the exact enzyme deficiency, a clue might be to note that the prefix "phenyl" in choice E is the same as in phenylketonuria.) Phenylalanine accumulation and its consequences lead to brain damage and mental retardation. The primary treatment for PKU is a diet low in phenylalanine (note that patients with PKU must avoid products with aspartame/Nutrasweet®, as it contains Phe).

Deficiency of alpha-keto acid decarboxylase (choice A), an enzyme involved in the catabolism of branched-chain amino acids (isoleucine, leucine, valine), results in maple syrup urine disease. Without this decarboxylase, the alpha-keto acids spill into the urine, giving it a characterisitc sweet smell.

Hexosaminidase A deficiency (choice B) results in Tay-Sachs disease. Cerebral accumulation of gangliosides results in mental retardation. Cherry red spots on retinal examination are characteristic.

Deficiency of homogentisate oxidase (choice C), an enzyme involved in tyrosine metabolism, results in alkaptonuria. In this disease, there is an accumulation of homogentisate derivatives that polymerize into melanin-like pigments. Classically, these patients will have brownish connective tissue and their urine will turn dark upon standing.

Near total deficiency of hypoxanthine-guanine phosphoribosyltransferase (choice D), an enzyme involved in the purine salvage pathway, results in Lesch-Nyhan syndrome. This syndrome is characterized by impaired purine salvage and increased de novo purine synthesis and catabolism, leading to hyperuricemia. Children with Lesch-Nyhan syndrome are mentally retarded and classically exhibit self-mutilating behavior.


 

 

Question 14:  GP


A 31-year-old stockbroker drives to a high-altitude mountain resort to do some rock-climbing. Later that day, he experiences headache, fatigue, dizziness, and nausea. Which point on the graph above best corresponds to the relationship between plasma bicarbonate, pH, and PCO2 in this patient?

A. Point A
B. Point B
C. Point C
D. Point D
E. Point E

 The correct answer is: B.

Description:
The correct answer is B. Acute mountain sickness is caused by hypoxemia and alkalosis due to exposure to high altitude. Hyperventilation occurs in response to the hypoxemia, which helps bring the oxygen saturation back toward normal, but "blows off" excessive amounts of CO2, producing acute respiratory alkalosis.

(Choice A) is respiratory acidosis with renal compensation or metabolic alkalosis with respiratory compensation.

(Choice C) is uncompensated respiratory acidosis.

(Choice D) is either metabolic acidosis with respiratory compensation or respiratory alkalosis with renal compensation.

(Choice E) represents normal values.

(Choice F) represents metabolic acidosis.

(Choice G) represents metabolic alkalosis.

(Choice H) represents partially compensated chronic respiratory acidosis.


 

 

Question 15: PATH H/L

A 13-year-old male presents to the emergency room with a deep skin abrasion on his knee. He states that it has not stopped bleeding since it happened during recess approximately 20-30 minutes ago. Physical examination reveals a well-developed, well-nourished adolescent. There are multiple purpura over his legs and arms, and a few scattered petechiae on his chest and gums. His bleeding time is 22 minutes, platelets = 300,000/mm3, hemoglobin = 11g/dL. A trial of cryoprecipitate transfusion does not improve his bleeding time. A normal platelet transfusion does improve bleeding time. Which of the following is the correct diagnosis?

A. Bernard-Soulier syndrome
B. Henoch-Schönlein purpura
C. Idiopathic thrombocytopenic purpura
D. Thrombotic thrombocytopenic purpura
E. Von Willebrand's disease

 The correct answer is: A.

Description:
The correct answer is A. Bernard-Soulier syndrome is an autosomal recessive disease of platelet adhesion which causes prolonged bleeding times in the presence of normal platelet counts. These patients' platelets cannot bind to subendothelial collagen properly because of a deficiency or dysfunction of the glycoprotein Ib-IX complex. Clinically the patients have impaired hemostasis and recurrent severe mucosal hemorrhage. The only treatment for an acute episode is a transfusion of normal platelets. This patient has a slightly decreased hemoglobin due to blood loss.
The underlying biochemical defect is the absent or decreased expression of the glycoprotein Ib/IX/V complex on the surface of the platelets. This complex is the receptor for von Willebrand factor (vWF), and the result of decreased expression is deficient binding of vWF to the platelet membrane at sites of vascular injury, resulting in defective platelet adhesion. This is demonstrated by the lack of aggregation of platelets in response to ristocetin, an antibiotic that normally causes platelets to aggregate. The end result is the lack of formation of the primary platelet plug and increased bleeding tendency. The cause of the thrombocytopenia is not definitely known, but it is probably related to a decreased platelet life span.
Henoch- Schönlein purpura (choice B) is a self-limited autoimmune vasculitis that affects children and young adults, usually following an upper respiratory infection. Affected individuals develop purpuric rashes on the extensor surfaces of their arms, legs, and buttocks. They also have abdominal pain and hematuria from glomerulonephritis. Despite the tendency toward hemorrhage, the bleeding times and platelet count would be normal.


gene GPIb(alpha). The most common form of BSS is a nonsense mutation where a STOP codon is wrongly inserted into the middle of a gene. This results in a much shortened protein. In the case of BSS, the mutation is a TGG (Tryptophan) codon to a TGA (STOP) codon. The resulting protein is severely shortened, with the entire transmembrane helix, and cytoplasmic domain absent (See Diagram 17.03.01). This results in a total lack of GPIb-complex on platelets, and leads to the aetiology outlined below.

There are a few known variants of BSS. These variants are mutations in the Leucine rich region of GPIb(alpha), which is thought to be important in binding to vWF (a factor important in clotting). Another variant is proposed to be a mutation in one of the other proteins making up the GPI-complex, a possible mutation in either GPIb(beta), GPV, or GPIX.
Idiopathic thrombocytopenic purpura (choice C) causes an increase in the bleeding time, but as the name implies, platelet counts are decreased.. There is bleeding from small vessels, especially of the skin, gastrointestinal tract and
Diagram 17.03.01genitourinary tract. Purpura and petechiae frequently develop. It is considered a self-limited autoimmune disorder, typically affecting children after a recent viral infection.

Thrombotic thrombocytopenic purpura (choice D) is characterized by an increased bleeding time, but a decreased platelet count. It is a rare disorder of unknown etiology, thought to be initiated by endothelial injury, which releases certain procoagulant materials into the circulation, causing platelet aggregation. It causes purpura, fever, renal failure, microangiopathic hemolytic anemia and microthrombi, generally in young women. In this disorder, platelet transfusion is actually contraindicated, as it can precipitate thrombosis.

Von Willebrand's disease (choice E) causes increased bleeding times with normal platelet counts. It is the most common inherited bleeding disorder, caused by a defect in von Willebrand factor, which aids the binding of platelets to collagen. Even though the platelets themselves are normal, binding is impaired, thus a platelet transfusion would not correct the problem. Cryoprecipitate, a plasma fraction rich in von Willebrand factor, would help in the case of von Willebrand's disease, but would not help with Bernard-Soulier syndrome.

How does the illness occur?

What causes the bleeding problem?

How is the diagnosis made?

Lab Studies:

1. Serum

thrombocytopenia

prolonged bleeding time

low glycoprotein Ib, IX, and V levels

platelets unresponsive to ristocetin in vitro

2. Peripheral Blood Smear

giant bizarre platelets (mean diameter = 5-6 microns)

Medical Care:

How is the illness treated?

In general, no medications are needed. Antifibrinolytic agents (eg, aminocaproic acid) may be useful for mucosal bleeding. For surgery or life-threatening hemorrhage, platelet transfusion is the only available therapy.

Desmopressin acetate (DDAVP) has been shown to shorten the bleeding time in some, but not all, patients with BSS. Recently, recombinant activated factor VII has been used to treat congenital platelet disorders.

Drug Category: Antifibrinolytics -- Used to enhance hemostasis when fibrinolysis contributes to bleeding.

Drug Name

Aminocaproic acid (Amicar) -- Inhibits fibrinolysis via inhibition of plasminogen activator substances and, to a lesser degree, through antiplasmin activity. The main problems are that the thrombi that form during treatment are not lysed and effectiveness is uncertain.

Adult Dose

30 g/d PO/IV in divided doses q3-6h; not to exceed 30 g/d

Pediatric Dose

Loading dose: 100-200 mg/kg PO/IV
Maintenance dose: 100 mg/kg/dose q4-6h; not to exceed 30 g/d

Contraindications

Documented hypersensitivity; evidence of active intravascular clotting process; because aminocaproic acid can be fatal in patients with disseminated intravascular coagulation (DIC), it is important to differentiate between hyperfibrinolysis and DIC

Interactions

Coadministration with estrogens may cause increase in clotting factors, leading to a hypercoagulable state

Pregnancy

C - Safety for use during pregnancy has not been established.

Precautions

Do not administer unless a definite diagnosis of hyperfibrinolysis has been made; caution with cardiac, hepatic, or renal disease; avoid rapid IV administration because this may induce hypotension, bradycardia, and/or arrhythmia

Drug Category: Vasopressin analogs -- Desmopressin stimulates factor VIII, prostaglandins, and plasminogen release, but the mechanism of action is not clear and may not be common to all 3 substances. Possesses effect on vessel walls that produces an increase in platelet adhesion. This local hemostatic action may account for its hemostatic properties.

Drug Name

Desmopressin acetate (DDAVP, Stimate) -- Used to decrease bleeding time in some, but not all, patients with BSS. It may be useful for minor bleeding episodes. The exact mechanism for this is unknown, but it may relate to increased levels of von Willebrand factor binding to some residual glycoprotein Ib in patients without an absolute deficiency.

Adult Dose

IV (DDAVP): 0.3 mcg/kg IV
Intranasal (Stimate 1.5 mg/mL):
<50 kg: 150 mcg (1 spray in one nostril) intranasally
>50 kg: 300 mcg (1 spray in each nostril) intranasally

Pediatric Dose

Administer as adults

Contraindications

Documented hypersensitivity; platelet-type von Willebrand disease

Interactions

Coadministration with demeclocycline and lithium decrease effects; fludrocortisone and chlorpropamide increase effects of desmopressin

Pregnancy

B - Usually safe but benefits must outweigh the risks.

Precautions

Avoid overhydration in patients using desmopressin to benefit from its hemostatic effects; restrict free water intake to avoid hyponatremia; mild facial flushing and headache are possible

Drug Category: Clotting factors -- Hemostasis is the physiological response to bleeding. Injury and factors released by platelets initiates the coagulation cascade, which is mediated by blood clotting factors. This results in formation of an insoluble fibrin clot, thus reinforcing the initial platelet plug. Clotting factors (ie, antihemophilic factor [factor VIII], factors VII or IX) function as cofactors in the blood coagulation cascade.

Drug Name

Antihemophilic factor, recombinant (Factor VII, Kogenate) -- Experience with the use of recombinant activated factor VII is limited in patients with congenital platelet disorders. Safety and efficacy still are being evaluated.

Adult Dose

70-110 mcg/kg IV

Pediatric Dose

Not established, limited data exist; administer as in adults

Contraindications

Documented hypersensitivity

Interactions

Not evaluated adequately

Pregnancy

C - Safety for use during pregnancy has not been established.

Precautions

Monitor patients for signs or symptoms of activation of the coagulation system or thrombosis; because of limited use, caution when used for prolonged dosing; risk of thrombotic adverse events after treatment is unknown


 

 

Question 16: BESCI GP

An 88-year-old male complaining of abdominal pain enters the emergency room with his wife. A mini-mental status exam reveals pronounced forgetfulness and confusion. The patient is discovered to have acute appendicitis requiring immediate surgery. He is unable to understand the situation and cannot provide informed consent. Which of the following further actions must the physician take?

A. Do not perform surgery
B. Have another doctor confirm the necessity of surgery
C. Obtain a court order to perform surgery
D. Obtain consent from his wife to perform surgery
E. Try to persuade the patient to consent to surgery

Correct, you answered D.

Description:
The correct answer is D. In cases in which an emergency exists, the patient is incompetent to give consent, and the withholding of treatment would be potentially life-threatening, the physician must seek out close relatives of the patient to supply consent. The physician should proceed with treatment, assuming the patient would want the treatment had he or she understood the situation.

Not performing surgery (choice A) could cost the patient's life.

Having another doctor confirm the necessity of surgery (choice B) is favorable (if done immediately) but not mandatory and does not change the patient's consent status.

Obtaining a court order (choice C) is not necessary with the patient's wife immediately accessible.

Trying to persuade the patient to consent to surgery (choice E) would not only waste time and prove futile but might agitate the patient as well.


 

 

Question 17: PHYS GI

A 42-year-old obese woman experiences episodic abdominal pain. She notes that the pain increases after the ingestion of a fatty meal. The action of which of the following hormones is responsible for the postprandial intensification of her symptoms?

A. Cholecystokinin
B. Gastrin
C. Pepsin
D. Secretin
E. Somatostatin

Correct, you answered A.

Description:
The correct answer is A. This woman has a risk profile (female, fat, forties) and symptomatology consistent with gallstones (cholelithiasis). As would be expected, contraction of the gallbladder following a fatty meal often exacerbates the pain caused by gallstones. Cholecystokinin (CCK), the release of which is stimulated by dietary fat, is the hormone responsible for stimulation of gallbladder contraction. It is produced in I cells of the duodenum and jejunum. In addition to gallbladder contraction, CCK also stimulates pancreatic enzyme secretion and decreases the rate of gastric emptying.

Gastrin (choice B) is produced by the G cells of the antrum and duodenum. Gastrin stimulates the secretion of HCl from the parietal cells and pepsinogen from the chief cells of the stomach. Gastrin secretion is stimulated by gastric distention, digestive products (e.g., amino acids), and vagal discharge.

Pepsin (choice C) is a protease produced by the chief cells of the stomach (as pepsinogen). It is involved in the digestion of proteins. Pepsinogen release is stimulated by vagal stimulation, gastrin, local acid production, secretin, CCK, and histamine.

Secretin (choice D) is produced by the S cells of the duodenum. It is secreted primarily in response to acidification of the duodenal mucosa. Secretin stimulates the secretion of bicarbonate-containing fluid from the pancreas and biliary ducts. This neutralization allows pancreatic enzymes to function. Secretin also inhibits gastric acid production and gastric emptying.

Somatostatin (choice E) is produced by the D cells of the pancreatic islets and in the gastric and intestinal mucosa. Somatostatin is an inhibitory hormone-it inhibits the secretion of most gastrointestinal hormones, gallbladder contraction, gastric acid and pepsinogen secretion, pancreatic and small intestinal fluid secretion, and both glucagon and insulin release.


 

 

Question 18: ANAT GP

Which of the pharyngeal pouches develops into the palatine tonsil?

A. First
B. Second
C. Third
D. Fourth
E. Fifth

 The correct answer is: B.

Description:
The correct answer is B. The epithelial lining of the second pharyngeal pouch buds into the mesenchyme to form the palatine tonsil. Part of the pouch remains in the adult as the tonsillar fossa.

It is important to review the other choices since pharyngeal pouch derivatives are typically tested on the USMLE Step 1:

The first pharyngeal pouch (choice A) develops into the middle ear cavity and eustachian tube.

The third pharyngeal pouch (choice C) develops into the thymus and the inferior parathyroid glands.

The fourth pharyngeal pouch (choice D) gives rise to the
superior parathyroid glands. Recall that abnormal development of the 3rd and 4th pouches leads to DiGeorge syndrome and results in hypocalcemia as well as abnormal cellular immunity and consequent susceptibility to viral and fungal illnesses.

The fifth pharyngeal pouch (choice E) gives rise to the C cells of the thyroid gland. These cells secrete calcitonin-a hormone that lowers serum calcium.

Endoderm lines the internal surface of pharyngeal arches and the outpouchings of the developing pharynx between these arches. There are four of these which are well defined pairs of pouches (Figure 12).

The first pharyngeal pouchgrows out between the first and second branchial arches. It's endoderm ultimately forms the lining of the auditory tube, tympanic cavity and mastoid antrum.

Endoderm of the second pharyngeal pouchforms the surface epithelium and lining of the crypts of the palatine tonsil. Mesenchyme surrounding the crypts differentiates into the rest of the tonsil.

Endoderm of the third pharyngeal pouchdifferentiates into inferior parathyroidsand the thymus. These structures then migrate to their respective locations in the adult.

The fourth pharyngeal pouchendoderm gives rise to superior parathyroid glandsand the ultimobranchial body.

BRANCHIAL (PHARYNGEAL) CYSTS AND FISTULAS

These cysts are formed by remnants of the pharyngeal pouches. They are usually located laterally in the neck along the anterior border of the sternocleidomastoid muscle. Occasionally these cysts open exteriorly on the surface of the neck. They can also open into the inside of the pharynx. When they do this, they form fistulas or small open canals. These fistulas drain laterally and are not found in the midline of the neck.

DEVELOPMENT OF THE THYROID AND THYROGLOSSAL DUCT CYSTS

Contrary to what you might think, the thyroid gland does not develop from pharyngeal pouches. It develops from a pit on the back of the tongue, the foramen cecum, and migrates down to its final location in the neck. During this migration it remains connected to the tongue by a narrow canal, the thyroglossal duct.

The thyroglossal duct normally disappears during development, but occasionally thyroglossal duct cysts can be found anywhere along the route of descent of the thyroid gland. Thyroglossal duct cysts are midlinestructures.

Aberrant thyroid tissue may also be found anywhere from the back of the tongue (foramen cecum) along the path of the migrating thyroid. The pyramidallobe of the thyroid is actually thyroid tissue that remains in the distal path of the descending thyroglossal duct.

ARCH

MUSCLES

NERVE

BONES, LIGS., CARTILAGES

 

 

 

First

temporalis, masseter,
med. & lat. pterygoids, ant. belly digastric, mylohyoid, tensor tympani, tensor palati

Trigeminal
(motor root)

(Meckel's Cart.) malleus, incus, sphenomandibular ligament

Second

muscles of facial
expression, post. belly
digastric, stapedius,
stylohyoid

Facial

(Reichert's Cart.)
stapes, styloid process
stylohyoid lig., upper part and lesser horn of the hyoid.

Third

stylopharyngeus

Glossopharyngeal

Lower part of hyoid and greater horn

Fourth & Sixth

most muscles of larynx, pharynx, soft palate

Cranial X (XI via X)

laryngeal cartilages


 

 

Question 19: PHARM NEURO

A worried mother complains to her pediatrician that both she and her 6-year-old son's teacher have noticed that the child has become inattentive.
She states that her son frequently stops what he is doing and "stares blankly into space" before resuming his activities. Electroencephalography reveals a 3/second spike and slow wave pattern of discharges. Which of the following agents would most effectively treat this child's disorder?

A. Carbamazepine
B. Diazepam
C. Ethosuximide
D. Methylphenidate
E. Phenytoin

 The correct answer is: C.

Description:
The correct answer is C. The child is suffering from absence (petit mal) seizures. The age of onset is typically from
3 to 7 years; seizures may continue into adolescence, but generally subside before adulthood. These seizures have been known to occur up to 100 times a day. This is a form of Epilepsyin which Seizuresare caused by abnormal neuronal discharge in the brain -- the "wires" of the brain's electrical circuitry get crossed and discharge abnormally).  Absence Seizuresare characterized by impairment of consciousness or posture, and usually last only a few seconds.  This type of seizure is more common in those under age 20.

  • Usually begin between ages 6 to 14 years and resolve by age 20

 

  • Sudden, brief, loss of consciousness (the child appears to "space out" for a few seconds)

 

  • Eye fluttering

 

  • Speech suddenly ceases for a few seconds

 

  • Chewing movements or lip smacking

 

  • Hand shaking

 

  • Loss of body muscle tone, and falling if standing

 

  • Urine incontinence

 

  • No confusion following a seizure

 

  • There is an atypical form of absence seizure that differs from the typical form in its duration (i.e., lasts longer), has a slower onset, slower recovery, and may be associated with confusion.

 

  • A child may have no memory in both forms of Absence Seizures.

 

Seizuresmay occur very frequently and then go unnoticed for months or years.

 

causes

  • Idiopathic (unknown)

 

  • Congenital (born with structural brain problem)

 

  • Perinatal (brain Injury at or near time of birth)

 

  • There may be a complication of liver, kidney, or other diseases.

 

diag

  • EEG (electroencephalograph) shows synchronous and symmetric 3-Hz spike-and-wave activity.

 

  • CT scan or MRI may be done to rule out a structural problem.

 

  • Blood tests to rule out kidney or liver infection may be needed if EEG and radiographs are normal.

 

trt

 

  • Ethosuximide

 

  • Valproic acid

 

  • Clonazepam

 

 

 

Similart condition

 

Complex partial Seizures

 

Febrile Seizures

 

 Ethosuximide is indicated for this type (but no other type) of seizure.

 Other drugs used in the treatment of absence seizures are valproic acid, clonazepam, and a new agent, lamotrigine.

Carbamazepine (choice A) is used in the treatment of tonic-clonic (grand mal) and partial (focal) seizures.

Diazepam (choice B) has long been the drug of choice for status epilepticus. Recently, lorazepam (a shorter acting benzodiazepine) has also been accepted as a drug for this condition. Intravenous phenytoin is used if prolonged therapy is required. Phenobarbital has also been used, especially in children. If the status epilepticus is very severe and does not respond to these measures, general anesthesia may be used.

Methylphenidate (Ritalin; choice D) is a stimulant used to treat children with attention deficit disorder. This child has no history of hyperactivity, and the underlying cause of his "inattentiveness" is his seizure disorder.

Phenytoin (choice E) is effective in all seizure types except for the one in this question (absence). Note that phenytoin has some idiosyncratic, test-worthy side effects, including hirsutism and gingival hyperplasia.

TYPES of seizures

 

·        Petit Mal (Absences)

·        Myoclonic and Clonic Seizures

·        Grand Mal Seizures (Tonic-Clonic)

·        Partial (Focal) seizure 

Petit Mal (Absences)

·         There are two types of Petit Mal, or absence : the typical and atypical. 

·         If the brain was damaged atypical absences will occur, if not, typical will take place. 

·         There is a short loss of consciousness, without spasms in a typical absence. The epileptic person stares in front of himself, pale-faced, the eye-lids trembling - he is absent. It is sometimes difficult to distinguish this from 'daydreaming' and straying in thought. Most of the times, an absence will be short of duration - about thirty seconds long. It often occurs with children and disappears relatively frequently before adulthood. 

·         Absences that are of a longer duration will affect the epileptic person in the end. They might result in a lack of coordination. The epileptic person might start to waddle, something might be dropped, the mouth might fall open, and the head might drop forward. Next, there is confusion and there is no recollection of what has preceded : retrograde amnesia. An absence that takes some time could eventually evolve into a Grand Mal seizure.

 

·         TYPICAL Muscleactivity changes No movement

·         Hand fumbling

·         Fluttering eyelids

·         Lip smacking

·         Chewing

·         Consciousness changesStaring episodes (unintentional)

·         Lack of awareness of surroundings

·         Sudden halt in conscious activity (movement, talking, etc.)

·         May be provoked by hyperventilationor flashing lights, in some cases

·         Abrupt beginning of seizure

·         Each seizurelasts no more than a few seconds

·         Full recovery of consciousness, no confusion

 

·         No memory of seizure

 

·         ATYPICAL Atonic seizure No muscle movement

·         Slumping, loss of posture

·         Loss of muscle tone

·         Falling down

 

 

·         Consciousness changes Unintentional staring

·         Lack of awareness of surroundings

·         Sudden stop of conscious activity (movement, talking, etc.)

·         Hand fumbling

·         Fluttering eyelids

·         May be provoked by hyperventilation, in some cases

·         May have slower, gradual beginning of seizure

·         Each lasts only seconds to minutes

·         Recovery may be slower

·         May have short period of confusion or bizarre behavior

 

 

·         No memory of seizure

 

·         Note: Unexplained difficulties in school and learning difficulties may be the first indication of petit mal seizures.

 

 

Myoclonic and Clonic Seizures

·         Almost every adult person has at one moment or other had the experience of a limb that was trembling or shaking during the sleep. This shaking does not immediately imply the presence of epilepsy. 

·         On the other hand, a very nervous tic to serious shaking might indicate the presence of epilepsy. These will sometimes be accompanied by absences and might be followed by seizures of Grand Mal.

Grand Mal Seizures (Tonic-Clonic)

·         There is a large misconception about epilepsy. Mostly, if you tell someone "I am epileptic", one will immediately think of the Grand Mal seizures. This kind is the most spectacular and consequently stands out the most. But seizures of Grand Mal, or Tonic Clonic, are not occurring that often. 

·         These seizures consist of two phases. They start with an aura : the epileptic person will be submitted to strange sensations. He might e.g. experience an odd feeling in the stomach, a bad taste, flashing lights, etc. An aura doesn't last long, seconds, but just long enough. It might just be long enough for the epileptic person to take the necessary precautions before the next phase to begin.

·         Unconsciousness will be instant as soon as the tonic phase starts. The muscles are contracted spasmodic during this phase of the Grand Mal seizure. The body will become stiff due to this, it will cause the epileptic person to fall down, because the leg muscles will also be tightened. 

·         The lungs are being pumped empty : the tightened muscles at the chest are pressing down the thorax and as a consequence breathing will be stopped momentarily. Due to lack of oxygen, the epileptic's face will turn blue-grey. There is an emission of breath which causes a cry. All muscles will be tense during the seizure. There is no pain during the seizure.

·         About twenty to thirty seconds afterwards, respiration will resume. This is the start of the clonic phase. The muscles are now continuously contracting and unwinding. The body will start to tremble first, followed by severe shocks. 

·         Fractures might occur due to the severity of the spasms made by the epileptic person. It also happens that the anal sphincter unwinds. But what is noticed more frequently are drooling and biting on one's tongue.

·         Finally, a few minutes later the frequency and the severity of spasms will diminish. Consciousness will soon be regained. There epileptic person might be feeling a bit weak or sleepy. The fall and the convulsions could have cause some headache or numbness in the limbs. There might also be some other injuries, but in general this turns better out than may be expected. 

·         It might occur that a one-sided paralysis follows a Grand Mal seizure - this condition can last up to 36 hours. The condition is better known as Todd's pareses and is often noticed with children. You might also observe a kind of long period of confusion: a kind of "twilight situation". The movements of the epileptic might be as if it concerns someone who is drunk and, if, encountered with opposition, a violent reaction can be expected.

 

 

 

Partial (Focal) seizure

·         There are two types: simple and complex partial seizures. 

·         Simple partial seizures. 

·         This type of seizures can be found under different shapes. The epileptic will experience strange perceptions : prickles, tingles, noises, strange sensations. There sometimes is a short paralysis. 

Jackson epilepsy. This type is an evolution of a spasm that started in e.g. the spasm. The seizure gradually extends itself at the side of the body where the spasm started.

Complex partial seizures.

With this type of seizure the epileptic also has an aura - a foreboding - and thus can take the time to take the necessary precautions. The seizure can consist of making grimaces, roaring with laughter, smacking one's lips, making swallow movements with one's mouth. 

About twenty to thirty seconds later these actions are followed by purposely wandering about and peculiar arm gestures. Sometimes this type of seizure might evolve into a Grand Mal seizure.

 

 

 

Anticonvulsant : ethosuximide, Zarontin

Generic Name: ethosuximide
Brand Name(s): Zarontin
Common Use: Anticonvulsant

Anticonvulsant

The control of absence (petit mal) epilepsy.

Contraindications
Ethosuximide should not be used in patients with a history of hypersensitivity to succinimides.

Precautions
Blood dyscrasias, including some with fatal outcome, have been reported to be associated with the use of ethosuximide; therefore, periodic blood counts should be performed. Ethosuximide is capable of producing morphological and functional changes in the animal liver. In humans, abnormal liver and renal function studies have been reported. Administer ethosuximide with extreme caution to patients with known liver or renal disease. Periodic urinalysis and liver function studies are advised for all patients receiving the drug. Cases of systemic lupus erythematosus have been reported with the use of ethosuximide. The physician should be alert to this possibility.

Ethosuximide may impair the mental and/or physical abilities required for the performance of potentially hazardous tasks, such as driving a motor vehicle or other such activity requiring alertness; therefore, caution the patient accordingly.
Ethosuximide, when used alone in mixed types of epilepsy, may increase the frequency of generalized tonic-clonic attacks in some patients.
Proceed slowly when increasing or decreasing dosage, as well as when adding or eliminating other medication. Abrupt withdrawal of anticonvulsant medication may precipitate petit mal status.

Adverse Side Effects

Gastrointestinal:
Gastrointestinal symptoms occur frequently and include anorexia, vague gastric upset, nausea and vomiting, cramps, epigastric and abdominal pain, weight loss, and diarrhea.

Hemopoietic:
Leukopenia, agranulocytosis, pancytopenia, aplastic anemia, eosinophilia.

Nervous system:
Neurologic and sensory reactions reported during therapy with ethosuximide have included drowsiness, headache, dizziness, euphoria, hiccups, irritability, hyperactivity, lethargy, fatigue, and ataxia. Psychiatric or psychological aberrations associated with ethosuximide administration have included disturbances of sleep, night terrors, inability to concentrate, and aggressiveness. These effects may be noted particularly in patients who have previously exhibited psychological abnormalities. There have been rare reports of paranoid psychosis, increased libido, and increased state of depression with overt suicidal intentions.

Integumentary:
Dermatologic manifestations which have occurred with the administration of ethosuximide have included urticaria, Stevens-Johnson syndrome, systemic lupus erythematosus, and pruritic erythematous rashes.

Genitourinary:
Microscopic hematuria.

Miscellaneous:
Myopia, vaginal bleeding, swelling of the tongue, gum hypertrophy, hirsutism.

 


 

 

Question 20: BIO GP


In the family shown above, individuals affected with profound deafness are represented by a shaded symbol.
The phenotypes of individuals in the fourth generation can best be explained by:

A. autosomal dominant inheritance
B. locus heterogeneity
C. mitochondrial inheritance
D. multifactorial inheritance
E. X-linked dominant inheritance

 The correct answer is: B.

Description:
The correct answer is B. Locus heterogeneity is the situation in which the same phenotype is caused by defects in different genes. It is commonly seen in syndromes resulting from failures in a complex pathway, such as hearing. In this example, locus heterogeneity is indicated because of complementation. In generations I through III, deafness appears to be due to a single autosomal recessive trait. In generation IV, deaf parents produce normal hearing children, suggesting that the parents have defects in different genes. The children are heterozygous at both loci, and so have normal hearing (complementation).

Autosomal dominant inheritance (choice A), would require multiple assumptions. In generation IV, the parents produce six normal children, although at best they are both heterozygous for the trait. The probability of that event, given that they are both heterozygous, is (1/4)6. Even more troublesome is the mating between II-1 and II-2, in which normal parents produce affected children. While it is possible to invoke the possibility that the gene is an autosomal dominant with incomplete penetrance, no data about this frequency is given, and this hypothesis still requires many more assumptions than the correct answer. Penetrance is the frequency at which the phenotype is expressed in a person who carries a particular genotype. It can complicate pedigree analysis.

Mitochondrial inheritance (choice C), would show a completely different pattern. Genetic information is transmitted to progeny on nuclear chromosomes as well as on mitochondrial chromosomes. Since mitochondria are inherited only from the mother, if affected she will give the trait to all of her progeny. A male with the trait will never transmit it to his progeny.

Multifactorial inheritance (choice D) is a nonmendelian inheritance pattern that depends upon the incremental contributions of multiple loci to a single phenotype, as well as upon contributions from environmental factors. Characteristics that exhibit a broad range of values, like height or eye color, are inherited in this manner.

X-linked dominant inheritance (choice E) affects either sex, but which of the children are affected depends upon the sex of the affected parent. If the male parent is affected, he will produce only affected daughters and normal sons, because he gives his X chromosome to all of his daughters and none of his sons. If the female parent is affected, she contributes an X chromosome to both daughters and sons, hence both can be affected. However, since she is almost always heterozygous for the trait, the probability is 50% that her children will receive a normal X chromosome. Remember that human females randomly inactivate one of their X chromosomes (
Lyon hypothesis). Due to this inactivation, a female who is heterozygous for an X-linked dominant will tend to be both more mildly and more variably affected then a male with the same trait.


 

 

Question 21: PHYSIO CV


At which point on the diagram above would the length of the myocardial sarcomere be approximately 2 mm with maximal actin-myosin cross bridging?

A. A
B. B
C. C
D. D
E. E

. The correct answer is: B.

Description:
The correct answer is B. The diagram of ventricular pressure in millimeters mercury (mmHg) against diastolic volume in milliliters (mL) is drawn as two curves: an intraventricular diastolic pressure curve (lower curve) and an intraventricular systolic pressure curve (upper curve). Point B is the diastolic volume at which initial stretch leads to maximal systolic contraction. This corresponds to the muscle length at which the sarcomeres are closest to their most effective length, Lmax, which is about 2.2 mm. As cardiac muscle is stretched, its ability to contract (after electrical stimulation) increases up to the muscle length corresponding to Lmax, after which it begins to decrease. Lmax represents the length at which potential cross-bridging between actin and myosin in each sarcomere is at a maximum. When applied to the entire heart, rather than a strip of isolated muscle, this length-tension relationship is responsible for the Frank-Starling law: increased end-diastolic volume increases force of systolic contraction (up to Lmax), after which peak systolic pressure begins to decrease.

Choices A, D, and E all correspond to minimum sarcomere length.

Choice C corresponds to an end diastolic pressure so high that no ejection is possible; end diastolic pressure and peak systolic pressure are the same.

 Law of Laplace: (P = 2HT / r)

A.As ventricular volume increases, ventricular circumference increases and lengths of individual cells increase B.At constant volume, increased intraventricular pressure causes increased tension in the individual cells of the ventricular muscle

C. As ventricular volume increases, a larger force from each muscle cell is required to produce any given intraventricular pressure

D. Pressure (P) = 2 x tension (T) x Wall thickness (H)/intraventricular radius (r)

1. Filling Phase (Fig.1, A-B):radius increases at constant filling pressure causing increased wall tension (stretch)

2. Isovolumetric Contraction (Fig.1, B-C):Tension increases at constant radius causing increased pressure

3. Ejection Phase (Fig.1, C-D):Radius decreases at constant tension causing further increase in pressure

4. Isovolumetric Relaxation Phase (Fig.1, D-A):Tension decreases at constant radius causing decreased pressure.

Excitation - Contraction Coupling:

A. Activation:1.Calcium couples electrical events to the subsequent mechanical events. 2.Contraction of cardiac muscle is absolutely dependent on the presence of extracellular as well as intracellular Ca2+ stores.

3.Transporters mediating movement of calcium into the cytosol.

a. The Ca channel and the SR-Ca release channel:i.)The Ca channel in heart is related to the DHP receptor in skeletal muscle (SKM). ii.)The cardiac Ca channel is not physically connected to the Ca-release channel in the SR.

iii.) Ca entering through the Ca channel induces opening of the SR Ca-release channel. This process is referred to as Ca-induced Ca-release.

 

b. The Na-Ca exchanger:

i.)Exchanges 1 Ca2+ for 3 Na+. ii.)The magnitude of the Na+ and Ca2+ gradients and the membrane potential (m.p.) determine the direction of net Ca flux, as mediated by the exchanger.

iii.)[Na+]o, [Na+]i, [Ca2+]o are constant on a beat-to-beat basis.

iv.)[Ca2+]i varies between 0.1 and 10 mM; membrane potential varies between -85 and +20mV.

v.)Exchanger mediates Ca2+ efflux at rest.

vi.)Exchanger mediates Ca2+ influx during early part of the action potential but net efflux as [Ca2+]i and m.p. change.

 

 

 

B. Relaxation:

1. SR Ca-ATPase sequesters Ca into the SR 2. Na-Ca exchanger mediates Ca2+ efflux

3. Sarcolemmal Ca-ATPase mediates Ca efflux

4.An amount of calcium equal to that which entered must exit the cell on a beat-to-beat basis if steady state is to be maintained. 

 

 

II. Determinants of Cardiac Contractile Force:

A. Contractility:

Contractility (force of contraction) is enhanced when the sensitivity of the filaments for calcium and/or calcium availability are enhanced. 1. Contractility and the Autonomic Nervous system:a. Sympathetic: Norepinephrine and epinephrine enhance Ca2+ entry through Ca-channels and Ca-ATPase activity causing enhanced calcium cycling. These neurotransmitters also alter heart rate (see below). b. Parasympathetic: ACh decreases Ca entry through Ca channels by counteracting the processes underlying sympathetic effects. These neurotransmitters also alter heart rate (see below).

 

2. Heart rate:

a.An increase in heart rate leads to an increase in calcium content of the SR. This enhances Ca-induced Ca-release, calcium availability, and contractile force (positive staircase, Bowditch effect). b.A decrease in heart rate leads to a decrease in calcium content of the SR, which decreases Ca-induced Ca-release, calcium availability, and contractile force.

 

3. Cardiac Glycosides(digoxin, digitoxin, ouabain, digitalis) inhibit the Na,K ATPase. This causes the transmembrane Na gradient to decrease, which diminishes the ability of the Na-Ca exchanger to pump calcium out of the cell. The net effect is an increase in calcium content of the SR and an increase in contractile force.

 

B. Length-tension relationships - isometric relationships:

1. The basics of actin-myosin interaction in heart are comparable to SKM &endash; i.e. sarcomere length determines thick-thin filament interactions and therefore force production. 2. The "length" of cardiac muscle cells is determined by the volume of blood in the chambers of the heart, which in turn is determined by filling pressure (preload).

3.Since the force of contraction of the heart is typically measured as pressure, the length-tension relationship in heart is usually measured as a pressure-volume relationship.

4.The relationship between pressure and volume was first described by Frank and Starling and is now referred to as the Frank-Starling relationship or Starling's law of the heart.

 

 

III. Pressure Volume Loops:

A. During the cardiac cycle, volume is not constant therefore length is not constant B. The volume in the ventricle at the onset of ventricular contraction is the End Diastolic Volume (EDV). The pressure required to establish this volume is the preload. The relationship between preload and EDV corresponds to the "rest-tension curve".

C.The ventricle must develop sufficient pressure to exceed that in the aorta before ventricular volume can decrease during the cardiac cycle. Aortic pressure during ejection is the afterload. If no volume can be ejected because the heart cannot develop sufficient pressure to exceed the aortic pressure, then the heart contracts isometrically.

D. Left ventricular function throughout the cardiac cycle can be visualized as a pressure volume loop, which relates the two pressure volume relationships.

1.The volume in the ventricle at the end of diastole is the end diastolic volume (EDV). Shortly after ventricular contraction begins, the mitral valve closes and the ventricle contracts isovolumetrically until intraventricular pressure exceeds aortic pressure. 2.Ejection begins as pressure in the ventricle exceeds that of the aorta and the aortic valve opens. Pressure continues to rise early during ejection but then falls late in ejection (see Law of La Place).

3. When ventricular pressure falls below aortic pressure, the aortic valve closes and isovolumetric relaxation ensues.

4. When ventricular pressure falls below atrial pressure the mitral valve opens and the filling phase begins. Shortly after filling begins, the ventricular pressure-volume relationship intersects and travels along the "rest-tension" curve.

5.The heart operates on the ascending limb of the active tension relationship, which insures heterometric reserve (ability to contract harder if volume increases suddenly) and the capacity to match input and output on a beat to beat basis.


 

 

Question 22: BIO GP

Absence of which of the following enzymes would impair the rate-limiting step of glycogenolysis?

A. alpha-1,4-glucan transferase
B. Glycogen phosphorylase
C. Glycogen synthase
D. Phosphoglucomutase
E. UDP-glucose pyrophosphorylase

. The correct answer is: B.

Description:
The correct answer is B. The key to excelling in biochemistry on the USMLE is to master the most clinically important elements of metabolic pathways: rate-limiting steps, irreversible steps, steps involving enzymes affected by genetic diseases, etc. In this case, glycogen phosphorylase is the enzyme involved in the rate-limiting step of glycogenolysis:


Note that this enzyme is activated in response to the binding of glucagon to liver cell receptors or the binding of epinephrine to muscle receptors, via signal transduction.

 

Table 1. Glucose Estimation by GOD/POD

 

Time

mg/dL

Fasting

89

30 minutes

123

1 hour

145

2 hours

91

3 hours

83

4 hours

90

 

Table 2. Table of Enzymes

 

Enzyme

Reaction

Pathway

Comments

Glucokinase

Glucose + ATPGlucose 6-phosphate + ADP

Glycolysis

Found in liver only; higher activity at higher concentrations of glucose

Pyruvate kinase

PEP + ADPPyruvate + ATP

Glycolysis

Catalyzes last step of glycolysis; irreversible reaction. Enzyme activated by feed forward action fructose 1,6-bisphosphate; synthesis stimulated by insulin.

phosphofructokinase-1 (PFK-1)

Fructose 6-phosphate + ATPFructose 1, 6-bisphosphate + ADP

Glycolysis

Rate-limiting step of glycolysis; allosteric activation by fructose 2,6-bisphosphate produced by PFK-2, an enzyme activated by insulin

fructose 1,6-bisphosphatase (FBP-1)

Fructose 1,6-bisphosphateFructose 6-phosphate + Pi

Gluconeogenesis

Rate-limiting step of gluconeogenesis; allosterically inhibited by fructose 2,6-bisphosphate broken down by FBP-2, an enzyme activated by glucagon

glucose 6-phosphatase

Glucose 6-phosphateGlucose + Pi

Gluconeogenesis

Catalyzes the last step in gluconeogenesis; irreversible reaction

glycogen synthase

UDP-Glucose + GlycogenGlycogen + PPi

Glycogenesis

Rate-limiting step of glycogen synthesis; activated by insulin through dephosphorylation; inactivated by glucagon through phosphorylation; adds glucose units to existing glycogen but cannot build glycogen from scratch

glycogen phosphorylase

Glycogen (Glucose)n + PiGlycogen (Glucose)(n-1) + Glucose-1-phosphate

Glycogenolysis

Rate-limiting step of glycogen breakdown; activated by glucagon and epinephrine through phosphorylation

glycogen phosphorylase kinase

Glycogen phosphorylase + ATPGlycogen phosphorylase -- P + ADP

Glycogenolysis regulation in muscles

This enzyme converts the inactive (dephosphorylated) form of glycogen phosphorylase to the active (phosphorylated) form

phosphoglucomutase

Glucose-1-phosphate <—> Glucose-6-phosphate

Part of glycogenolysis

Important enzyme that forms glucose 6-phosphate from the breakdown of glycogen. Glucose 6-phosphate can then be used in glycolysis or converted to glucose by the action of glucose 6-phosphatase.

 



a-1,4-glucan transferase (choice A) is a debranching enzyme that removes three or four glucose units from a branch point and transfers them to the end of another chain. In this reaction, one alpha-1,4 bond is cleaved and another is formed. The elongated chain then becomes a substrate for glycogen phosphorylase.

Glycogen synthase (choice C) is the enzyme involved in the rate-limiting step of glycogen synthesis:


Note that this enzyme is inactivated in response to the binding of glucagon to liver cell receptors or the binding of epinephrine to muscle receptors, via signal transduction.

Phosphoglucomutase (choice D) is a key enzyme in glycogen synthesis that reversibly converts glucose-6-phosphate to glucose-1-phosphate.

UDP-glucose pyrophosphorylase (choice E) is another key enzyme in glycogen synthesis:


Note that the linkage formed between UDP and glucose is a high-energy bond that can provide energy to many biosynthetic reactions.

Reactions of Glycolysis 

First step of glycolysis is glucose transport (facilitated diffusion) across the sarcolemma that is accomplished by a specific protein on the plasma membrane–requires insulin during resting states.

1. phosphoryl transfer (phosphate group attached to the sixth carbon)

 

ATP-Mg2+ ––>  ADP

 

glucose

—————————>

glucose 6-phosphate

 

hexokinase

 

2. isomerization

glucose-6-phosphate

—————————>

fructose 6-phosphate

 

3. phosphoryl transfer (phosphate group attached to the first carbon)

 

ATP- Mg2+ ––>  ADP

 

fructose 6-phosphate

—————————>

fructose 1,6-diphosphate

 

phosphofructokinase

 

4. splitting of six-carbon compound into two three-carbon compounds (phosphate group is attached to third carbon of each compound)  

fructose 1,6-diphosphate

—————————>

2 glyceraldehyde 3-phosphate

5. reduction of NAD+

 

Pi + NAD+ ––>  NADH + H+

 

glyceraldehyde 3-phosphate

—————————>

1,3-diphosphoglycerate

6. formation of ATP

 

ADP + Pi ––>  ATP

 

1,3-diphosphoglycerate

—————————>

3-phosphoglycerate

7. phosphoryl shift

3-phosphoglycerate

—————————>

2-phosphoglycerate

8. dehydration (release of H2O)

2-phosphoglycerate

—————————>

phosphoenolpyruvate + H2O

9. formation of ATP

 

ADP + Pi ––>  ATP

 

phosphoenolpyruvate

—————————>

pyruvate

 

pyruvate kinase

 

10. reduction of pyruvate (two possibilities)

 

NADH + H+ ––>  NAD+

 

a) pyruvate

—————————>

lactate

 

lactate dehydrogenase

 

 

NAD+ ––> NADH + H+

 

b) pyruvate + Coenzyme A

—————————>

acetyl Coenzyme A + CO2

 

pyruvate dehydrogenase

 

 

Glycogenolysis

 

phosphorylase

 

[glucose]n + Pi

<—————————>

glucose 1-P + [glucose]n-1

 

glucose 1-P

—————————>

glucose 6-P

 

 Regulation of glycogenolysis 

 

+ cAMP, Ca2+, ADP, AMP, IMP, Pi

 

phosphorylase b (inactive)

<—————————————>

phosphorylase a (active)

 

-- ATP,G 6-P, H+

 

 

Glycogenesis

glucose 6-P

<—————————>

glucose 1-P

glucose 1-P

—————————>

UDP-glucose

 

 

glycogen synthase (active)

 

UDP-glucose + [glucose]n

<—————————>

UDP + [glucose]n+1

 

Regulation of glycogenesis

 

+ cAMP, Ca2+

 

glycogen synthase (active)

<—————————>

glycogen synthase (inactive)

 

-- insulin

 

 

Regulation of Glycolysis 

Adenine nucleotide energy charge =

[ADP] + 2 [ATP]

X 0.5

 

[AMP] + [ADP] + [ATP]

 

 

 

·         if all adenine is in the form of ATP, the energy charge is 1.0; if all adenine is in the form of AMP, the energy charge is 0.0; energy charge is usually ~0.8

Phosphofructokinase (PFK)

Stimulators

Inhibitors

ADP

ATP

Pi

CP

AMP

H+

NH4+

citrate

increased temperature

 

 

Lactate dehydrogenase (LDH)

Hexokinase

 

 


 

 

Question 23: BESCI GP

A 34-year-old female is brought to her family physician by her brother.
He states that the patient's husband had run away with another woman the previous day. After discovering this, the patient arrived at her brother's home in a "dazed state," was unable to function appropriately the remainder of the evening, and slept fitfully all night. Early in the morning she began to talk to people who were not present, and debated whether or not to kill her children and herself. He denies anything like this ever happening before. Which of the following is the most likely diagnosis?

A. Bipolar disorder
B. Brief psychotic disorder
C. Schizoaffective disorder
D. Schizophrenic disorder
E. Schizophreniform disorder

 The correct answer is: B.

Description:
The correct answer is B. Brief psychotic disorder is a thought disorder of psychotic proportion (e.g., the demonstrated auditory hallucinations). There is often a precipitating event. To make this diagnosis, the patient's symptoms must have been present for at least 1 day but not more than 1 month. This duration distinguishes brief psychotic disorder from schizophrenic and schizophreniform disorders.

Brief Psychotic Disorder Brief Psychotic Disorder is a psychosis that has a rapid onset, generally following a major stressor. Its hallmark is emotional turmoil, mood lability, and confusion. The sufferer must have one of the following symptoms: delusions, hallucinations, grossly disorganized or catatonic behavior, or disorganized speech. A brief psychotic episode is time limited, lasting at least a day, but less than a month. As a rule, the quicker the onset, the faster the recovery. acute psychotic episode lasting longer than one day but less than one month

SYMPTOMS

Presence of one (or more) of the following symptoms:

(a) delusions
(b) hallucinations
(c) disorganized speech (e.g., frequent derailment or incoherence)
(d) grossly disorganized or catatonic behavior

Note: Do not include a symptom if it is a culturally sanctioned response pattern.

Duration of an episode of the disturbance is at least 1 day but less than 1 month, with eventual full return to premorbid level of functioning.

The disturbance is not better accounted for by a Mood Disorder With Psychotic Features, Schizoaffective Disorder, or Schizophrenia and is not due to the direct physiological effects of a substance (e.g., a drug of abuse, a medication) or a general medical condition.

 



Bipolar disorder (choice A) is defined by the presence of a manic episode (characterized by an altered, intense mood, with rapid/pressured speech and hyperactivity).

Schizoaffective disorder (choice C) is characterized by elements of both schizophrenia (e.g., psychotic symptoms) and mood disorders (e.g., manic or depressive symptoms).

Schizophrenic disorder (choice D) is a chronic recurrent thought disorder often accompanied by bizarre behaviors. It usually begins in early adulthood. To make this diagnosis, the symptoms must last 6 months or more, which distinguishes it from brief psychotic and schizophreniform disorders.

Schizophreniform disorder (choice E) is a thought disorder of psychotic proportion with a duration between 1 and 6 months. This duration distinguishes the condition from brief psychotic disorder and schizophrenic disorder. Often, the onset of the thought disorder is as a young adult, when the person must leave the protected environment of the home and begin to assume personal responsibility for his or her life. He or she is unable to cope with the world and his/her defenses deteriorate to the point of psychosis. Typically, patients with schizophreniform disorder were unable to maintain interpersonal relationships as adolescents.

Old term: Brief Reactive Psychosis

Associated Features

Diagnostic Criteria

  1. Presence of one (or more) of the following symptoms:
    1. delusions
    2. hallucinations
    3. disorganized speech (e.g., frequent derailment or incoherence)
    4. grossly disorganized or catatonic behavior

 

  1. Duration of an episode of the disturbance is at least 1 day but less than 1 month, with eventual full return to premorbid level of functioning.
  2. The disturbance is not better accounted for by a Mood Disorder With Psychotic Features, Schizoaffective Disorder, or Schizophrenia and is not due to the direct physiological effects of a substance (e.g., a drug of abuse, a medication) or a general medical condition.

·          With Marked Stressor(s) (brief reactive psychosis): if symptoms occur shortly after and apparently in response to events that, singly or together, would be markedly stressful to almost anyone in similar circumstances in the person's culture

·         Without Marked Stressor(s): if psychotic symptoms do not occur shortly after, or are not apparently in response to events that, singly or together, would be markedly stressful to almost anyone in similar circumstances in the person's culture

·         With Postpartum Onset: if onset within 4 weeks postpartum

Differential Diagnosis

 

Some disorders have similar or even the same symptom. The clinician, therefore, in his diagnostic attempt has to differentiate against the following disorders which he needs to rule out to establish a precise diagnosis.

 

·         Psychotic Disorder Due to a General Medical Condition or a Delirium;

·        

·         Substance-Induced Psychotic Disorder;

·         Substance-Induced Delirium and Substance Intoxication;

·         Mood Episode;

·         Schizophreniform Disorder;

·         Delusional Disorder;

·         Mood DisorderWithPsychotic Features;

·         Psychotic Disorder Not Otherwise Specified;

·         Factitious Disorder, With Predominantly Psychological Signs and Symptoms;

·         Malingering;

·         Personality Disorders.

 

 

Antipsychotics
(typical)

Dose
mg/day

Anti-
cholineric
¹

EPS²

Sedation

Hypotension
(orthostatic)

Chlorpromazine
(Thorazine)

100-2000

++++

++

+++++

+++++

Thioridazine
(Mellaril)

100-600

+++++

+

++++

+++++

Trifluoperazine
(Stelazine)

5-60

++

++++

+

++

Thiothixene
(Navane)

5-60

++

++++

++

++

Fluphenazine
(Prolixin)

5-30

++

+++++

++

++

Haloperidol
(Haldol)

2-200

+

+++++

++

+

 

Antipsychotics
(atypical)

Dose
mg/day

Anti-
cholineric
¹

EPS²

Sedation

Hypotension
(orthostatic)

Risperidone
(Risperdal)

1-6

+

+

+

++

Olanzapine
(Zyprexa)

5-20

+

+

++

+

Clozapine³
(Clozaril)

5-60

+++++

+

+++++

+++++

 

¹ Dry mouth, constipation, blurred vision, urinary retention.

² Extrapyramidal side effects (dystonia, parkinsonism, akathisia, tardive dyskinesia).

³ Requires weekly White Blood Cell (WBC) count because of risk of agranulocytosis.

Adapted from Bernstein JG: Handbook of drug therapy in psychiatry, ed 3, St. Louis, 1995, Mosby.

 

 Special Antipsychotic Adverse Reactions  

 

  1. Neuroleptic Malignant Syndrome. May occur at any point during the course of treatment. Includes symptoms of autonomic instability, altered mental status, which may progress to hyperthermia, stupor, and muscle hypertonicity. Death may occur. Cause: Neuroleptics (phenothiazines, etc.) Characteristics. Same symptoms as malignant hyperthermia (see below) but generally develops over days instead of minutes. Treatment: As per malignant hyperthermia.
    • Malignant Hyperthermia. Cause. 1:20,000 in response to a muscle-relaxing agent (such as succinylcholine) or an inhaled anesthetic (such as halothane). Is hereditary. May also be secondary to physical or emotional stress. Characteristics. Hyperthermia, muscle rigidity, tachycardia, acidosis, shock, coma, rhabdomyolysis. Treatment includes IV dantrolene 1 to 10 mg/kg IV titrated to effect, management of acidosis and shock, peripheral cooling (see management of heat stroke below).

 

  1. Tardive Dyskinesia. Involuntary movements of the tongue, face, mouth, or jaw associated with long-term administration of antipsychotics. Elderly females at highest risk. May be irreversible.

 Schizophreniform Disorder

For a material part of at least one month (or less, if effectively treated) the patient has had 2 or more of:

Delusions (only one symptom is required if a delusion is bizarre, such as being abducted in a space ship from
      the sun).

Hallucinations (only one symptom is required if hallucinations are of at least two voices talking to one another or of a voice that keeps up a running commentary on the patient's thoughts or actions)

Speech that shows incoherence, derailment or other disorganization


Severely disorganized or catatonic behavior.

Any negative symptom such as flat affect, muteness, lack of volition.

Including prodromal, active and residual phases, an episode of the illness has lasted at least one month but not longer than 6 months.

Mood and schizoaffective disorders have been ruled out, because the duration of any depressive or manic episodes that have occurred during the psychotic phase has been brief.

This disorder is not the direct physiological result of a general medical condition or the use of substances, including prescription medications.

A statement of prognosis should be added to the diagnosis:

With Good Prognostic Features (2 or more of the following):

Actual psychotic features begin within 4 weeks of the first noticeable change in the patient's functioning or behavior.
The patient is confused or perplexed when most psychotic.
Premorbid social and job functioning are good.
Affect is neither blunt nor flattened.

Associated Features:

Learning Problem
Hypoactivity
Psychosis
Euphoric Mood
Depressed Mood
Somatic or Sexual Dysfunction
Hyperactivity
Guilt or Obsession
Sexually Deviant Behavior
Odd/Eccentric or Suspicious Personality
Anxious or Fearful or Dependent Personality
Dramatic or Erratic or Antisocial Personality

Differential Diagnosis:

Some disorders have similar or even the same symptom. The clinician, therefore, in his diagnostic attempt has to differentiate against the following disorders which he needs to rule out to establish a precise diagnosis.

Schizophrenia;
Brief Psychotic Disorder.

Cause:

This diagnosis is used when the initial signs of schizophrenia are present. Psychotic symptoms can dissipate after a short period and never return for some people And because significant stressors and substance use can bring about psychotic symptoms, this diagnosis is used for the first six months. If symptoms persist after six months, then a diagnosis of Schizophrenia is given.

Schizophreniform disorder appears to be related to abnormalities in the structure and chemistry of the brain, and appears to have strong genetic links; but its course and severity can be altered by social factors such as stress or a lack of support within the family. The cause of schizoaffective disorder is less clear cut, but biological factors are also suspected

Treatment:

Medication is the most important part of treatment as it can reduce and sometimes eliminate the psychotic symptoms. Case management is often needed to assist with daily living skills, financial matters, and housing, and therapy can help the individual learn better coping skills and improve social and occupational skills

 


 

 

Question 24: PATH REPRO

A 34-year-old woman presents with abdominal pain and reports changes in her bowel habits. On pelvic examination a mass is palpated on her right ovary. Laparoscopic surgery is performed and the mass is removed. The gross appearance of the tumor is shown below. What type of tumor is it?

these are the type of picture seen during the usmle where nothing is certain and too dark





A. Choriocarcinoma
B. Dysgerminoma
C. Fibroma
D. Granulosa-theca cell
E. Teratoma

 The correct answer is: E.

Description:
The correct answer is E. The most important clue in this question is the photograph. The presence of skin, teeth, and hair in the tumor should have led you to the diagnosis of dermoid cyst, also known as a mature teratoma. These tumors are a benign type of germ cell tumor. They are lined by stratified squamous epithelium and skin adnexal structures (sebaceous glands with associated secretions and hair follicles). They occur most commonly in women of reproductive age.

Choriocarcinomas (choice A) are a malignant type of germ cell tumor that are often widely metastatic. An important clue to this type of tumor is an elevated level of hCG.

Dysgerminomas (choice B) are also germ cell tumors. They are rare and usually occur in women in their 20s and 30s. They are composed of sheets of large cells with clear cytoplasm and are notable for their high degree of radiosensitivity.

Ovarian fibromas (also called fibroma-thecomas; choice C) are a type of sex cord-stromal tumor. They present with pain and a pelvic mass. Ascites is common and hydrothorax may occur. The triad of ovarian mass, ascites, and hydrothorax is called Meig's syndrome and most commonly occurs with fibroma.

Granulosa-theca cell tumors (choice D) are a type of sex cord-stromal tumor that typically occur in post-menopausal women. These tumors contain a variable ratio of granulosa cells (small polygonal cells in sheets or strands) and theca cells (cuboidal/spindle-shaped). The granulosa cells may be filled with acidophilic material known as Call-Exner bodies. These tumors may secrete estrogens and are associated with endometrial and breast cancer.

 

OVARIAN TERATOMA

Germ cell tumors represent 15-20% of ovarian tumors. Germ cell tumors consist of teratomas, dysgerminoma, endodermal sinus tumor, choriocarcinoma, embryonal carcinoma, polyembryoma and mixed germ cell tumors.

Hitogenesis of the ovarian germ cell tumors

DISTINCTIVE FEATURES

Appear in three forms. Mature (benign) teratomas (dermoid cyst), Immature (malignant) teratomas, and monodermal or specialized teratomas.

MATURE (BENIGN) TERATOMA:

Benign teratomas are bilateral in 10-15% of cases.
These tumors are cystic, are relatively small and consist of cysts lined by epidermis and adnexal structures.
The cyst may be filled with hair, sebaceous material, and teeth.
The cyst wall may contain differentiatated tissues including bone, cartilage, muscle, thyroid follicles, lining of the gastrointestinal tract, respiratory tract and other tissues.

IMMATURE (MALIGNANT) TERATOMA:

Rare tumors.
Most common in adolescents and young women.
Most of these types of tumors are malignant, grow quickly and metastasize widely.
Grossly, appear as large solid tumors with areas of necrosis and hemorrhage.
Microscopically consist of embryonic cells from one to three germ cell layers. May contain immature elements differentiating to bone, cartilage, epithelium, muscle, nerve and other tissues. The tumor is classified as grade I-III depending on the extent of the immaturity of the various elements and presence of neuroepithelium.

MONODERMAL (SPECIALIZED) TERATOMA:

The cells in these tumors grow along a single germ cell layer. Examples include struma ovarii which consists of mature thyroid tissue and ovarian carcinoid.

 

 


Question 25: PHARM GP

A 55-year-old woman is receiving chemotherapy for non-Hodgkin's lymphoma.
Several days after a treatment, she notes that she has blood in her urine. Which of the following antineoplastic drugs is most likely responsible for this side effect?

A. Bleomycin
B. Cisplatin
C. Cyclophosphamide
D. Doxorubicin
E. Plicamycin

 The correct answer is: C.

Description:
The correct answer is C. Step 1 questions related to antineoplastic agents are most likely going to ask you about mechanism of action, cell cycle specificity, or side effects. While bone marrow suppression, nausea, vomiting, ulcers, and alopecia are common side effects, you are probably more likely to be asked about the unique side effects of certain drugs. In this case, cyclophosphamide has the unique side effect of hemorrhagic cystitis, which could cause hematuria. Note that cyclophosphamide is also consistent with the fact that the patient is being treated for non-Hodgkin's lymphoma.

All the other answer choices have unique, USMLE-worthy side effects:

Bleomycin (choice A
) is notable for its pulmonary toxicity. Note that while bleomycin can be used to treat lymphoma, you are more likely to see it as part of the treatment regimen (with vinblastine and cisplatin) for testicular tumors.

Cisplatin (choice B) is notable for its
nephrotoxicity and ototoxicity. It is used in the treatment of testicular and lung cancers. It would not be used in a patient with lymphoma.

Doxorubicin (choice D)
Its unique side effect is its cardiotoxicity.is also called adriamycin. It is an antibiotic agent used in the treatment of many cancers, including lymphomas, sarcomas, and many carcinomas.

You might have been tempted by plicamycin (choice E) if you recalled that it can cause a hemorrhagic diathesis. You could have eliminated this answer, however, because
plicamycin is not used in the treatment of lymphoma. It is used in the treatment of testicular cancer and cases of hypercalcemia.

Plicamycin(Mithramycin)

 

 

Mechanism of action:binds to DNA -- interrupts DNA-directed RNA synthesis

Also decreases plasma calcium (independent tumor cell action;acts on osteoclasts)

 

 

 

 

Clinical Uses:

some efficacy in testicular cancer that is unresponsive to standard treatment:

 

especially useful in managing severe hypercalcemia associated with cancer

 

 

 

 

 Adverse Effects:

nausea

 

vomiting

 

thrombocytopenia

 

leukopenia

 

hypocalcemia

 

liver toxicity

 

bleeding disorders